Módulo 1 Psiquiatra Flashcards

1
Q

A 39-year-old African American man is an inpatient on a psychiatric ward. He was admitted because of concerns that his neighbors are spying on him and devising ways to kill him. He states that the neighbors have inserted cameras in several rooms of his house to monitor his activities. He claims to hear them through the walls saying they are going to kill him. The patient’s wife called the police when he bought a gun stating that he was going to kill the neighbors “in self-defense.” Which of the following is the most appropriate pharmacologic treatment?

(A) Benztropine
(B) Diazepam
(C) Fluoxetine
(D) Lithium
(E) Risperidone

A

Respuesta: E

The correct answer is E. This clinical vignette illustrates an individual with a primary psychotic disorder, probably schizophrenia. Risperidone, an atypical antipsychotic, would treat symptoms of paranoid delusions and auditory hallucinations.

Benztropine (Choice A) is an anticholinergic medication that is generally used with the typical antipsychotics (such as haloperidol) for the prophylaxis of extrapyramidal symptoms. It has no antipsychotic properties.

Diazepam (Choice B) is a long-acting benzodiazepine that would be appropriate in the management of an anxiety disorder. Shorter-acting benzodiazepines, such as lorazepam, are often used adjunctively in psychotic individuals for acute agitation.

Fluoxetine (Choice C) is a selective serotonin reuptake inhibitor (SSRI) used in depressive illnesses.

Lithium (Choice D) is used in bipolar disorder. Although individuals with bipolar disorder may have delusions, they are usually of a grandiose quality. In such individuals, antipsychotic medications like risperidone are often given as well, because lithium takes approximately 10 days to have a beneficial effect. This patient has profound paranoid delusions, suggestive of schizophrenia.

How well did you know this?
1
Not at all
2
3
4
5
Perfectly
2
Q

A 61-year-old man is brought to a physician for evaluation of his behavior. His family states that recently he has been yelling, spitting, and pulling other people’s noses or ears unexpectedly. He was always a fine man and devoted husband but now uses foul language and is openly promiscuous. He has no significant medical or psychiatric history. The family recalls that the patient’s uncle displayed similar behavior and was placed in a nursing home, where he quickly died. On examination, memory and visuospatial functions are intact, but the patient exhibits some word-finding difficulties. During the examination, the patient keeps repeating the doctor’s command and giggling inappropriately and loudly. Which of the following is the most likely diagnosis?

(A) Alzheimer disease
(B) Dementia pugilistica
(C) Multi-infarct dementia
(D) Neurosyphilis
(E) Pick disease

A

Respuesta: E

The correct answer is E. Pick disease is one in the spectrum of frontotemporal dementias. Unlike most other dementias, which present initially with cognitive changes, Pick disease presents insidiously with behavioral changes related to atrophy in frontotemporal regions. The cause is unknown. It is more common in men, especially those with an affected first-degree relative. In the early stages, it is more often characterized by personality and behavioral changes, such as disinhibition, impulsivity, repetitive behaviors, hypersexuality, and hyperorality. Treatment relies on behavior management.

Alzheimer disease (Choice A) is a progressive dementia with associated risk factors that include age, positive family history, head trauma, and Down syndrome. Typical first symptoms are a subtle loss of short-term memory, language difficulties, and apraxias, followed by impaired judgment and personality changes. Psychiatric symptoms are often prominent in the course of the illness. Treatment targets specific symptoms and includes psychosocial interventions and pharmacologic therapy.

Dementia pugilistica (Choice B) is a posttraumatic dementia that develops after blunt head trauma. It often occurs as a result of motor vehicle accident injuries and sports-related traumas. The clinical symptoms are dependent on the areas affected most (cortical versus subcortical). After a period of amnesia and recovery, the most common symptoms are decreased attention, slowed information processing, increased distractibility, and problems with memory. Behavioral changes include impulsivity, depression, aggression, and personality changes. Treatment is targeted at controlling the symptoms.

Multi-infarct dementia (Choice C), or vascular dementia, accounts for 20% of the cases of dementia. Typical features include progression of cognitive deficits and associated motor or sensory neurologic deficits. Risk factors are associated with vascular disease, vasculitis, or embolic disease. Treatment is focused on addressing risk factors.

Neurosyphilis (Choice D) is a dementia of infectious origin, which is potentially reversible if diagnosed and treated early. It appears 10-15 years after the primary infection. It generally affects the frontal lobes, resulting in personality changes, poor judgment, irritability, and decreased care for oneself. Delusions of grandeur are seen in 10% to 20% of affected patients. The disease progresses into dementia with neurologic symptoms, such as tremor, Argyll-Robertson pupils, dysarthria, and hyperreflexia. Cerebrospinal fluid tests confirm the diagnosis. Treatment targets the infection and includes administration of IV penicillin G.

How well did you know this?
1
Not at all
2
3
4
5
Perfectly
3
Q

While on the Psychiatry Consult-Liaison Inpatient Service, a psychiatry intern is called to assess a patient on a general medical floor who has developed a muscle spasm causing her neck to twist uncontrollably to the left. She is also having difficulty speaking and is upset. The intern evaluates the patient’s list of medications and concludes that the her new symptoms may be due to one of them. Which of the following medications is most likely responsible for the patient’s symptoms?

(A) Aspirin
(B) Digoxin
(C) Erythromycin
(D) Fluoxetine
(E) Metoclopramide

A

Respuesta: E

The correct answer is E. Metoclopramide is used as a gastric motility agent, often in patients with diabetes who have gastric paresis. It has antidopaminergic properties and can cause acute dystonic reactions such as are occurring in this patient. A dystonia is a spontaneous contraction of individual muscles. Treatment includes cessation of the metoclopramide and providing an anticholinergic agent, such as benztropine, or an antihistamine, such as diphenhydramine, both of which are usually given in IM form for immediate effect.

Aspirin (choice A) is an analgesic, antipyretic, anti-inflammatory, and antiplatelet agent. It is widely used and does not cause acute dystonic reactions.

Digoxin (choice B) is a cardiac medication, specifically a steroid glycoside, used in the treatment of certain heart diseases, especially congestive heart failure. It does not cause acute dystonic reactions.

Erythromycin (choice C) is a macrolide antibiotic and does not cause acute dystonic reactions.

Fluoxetine (choice D) is an antidepressant medication. It is a selective serotonin reuptake inhibitor and has not generally been associated with acute dystonic reactions.

How well did you know this?
1
Not at all
2
3
4
5
Perfectly
4
Q

A 29-year-old woman was attacked, held at gunpoint, robbed, and beaten after leaving a restaurant in the evening. Despite this, she managed to report the incident to police and continue with her daily activities. Two months later, she seeks psychiatric help because she has been having difficulty going to work and participating in other activities. Which of the following constellation of symptoms would she most likely report to her psychiatrist?

(A) Confusion and disorientation
(B) Depression and suicidal thoughts
(C) Euphoria and racing thoughts
(D) Flashbacks and increased arousal
(E) Hyperphagia and hypersomnia

A

Respuesta: D

The correct answer is D. The patient is experiencing symptoms of acute posttraumatic stress disorder. The event is persistently reexperienced in flashbacks, nightmares, increased arousal, and avoidance of stimuli associated with trauma. The symptoms are of less than 3 months’ duration for the acute course, and the disturbances cause significant social or occupational impairment.

Confusion and disorientation (choice A) are symptoms usually seen in cognitive disorders (delirium, dementia) but can also be seen as a part of psychotic disorders.

Depression and suicidal thoughts (choice B) are symptoms seen in various mood disorders and in the depression associated with psychosis. They can also be associated with anxiety disorders and substance-induced mood disorders.

Euphoria and racing thoughts (choice C) are usually symptoms of bipolar disorder (manic or mixed type) or schizoaffective disorder. These symptoms can also be seen with substance abuse.

Hyperphagia and hypersomnia (choice E) are symptoms of major depressive disorder with atypical features, along with sensitivity to rejection and a heavy leaden feeling in limbs. They can also be symptoms of several medical conditions.

How well did you know this?
1
Not at all
2
3
4
5
Perfectly
5
Q

A 25-year-old man presents to the emergency department (ED) with a sore arm and difficulty using his hand. He has been evaluated multiple times in the ED for multiple somatic complaints, but medical evaluation has been consistently unrevealing. He also has a history of cocaine abuse, for which he was admitted to various substance abuse rehabilitation facilities. Often, his trips to the hospital coincided with scheduled court appearances after violating probation. Physical examination of the man’s arm does not show obvious injury or focal abnormalities, and an x-ray of the arm is normal. A urine toxicology screen is negative. The patient was offered ice, anti-inflammatory medication, and information concerning the benign nature of his pain. The patient was then told that he was to be discharged from the ED, at which point he stated that his pain was too great for him to leave the hospital and he demanded admission. Which of the following is the most likely diagnosis?

(A) Conversion disorder
(B) Dissociative identity disorder
(C) Factitious disorder
(D) Malingering
(E) Rheumatoid arthritis

A

Respuesta: D

The correct answer is D. This patient is malingering. He appears to be intentionally producing symptoms of arm pain in an attempt to gain admission to the hospital. His pattern has been such that his behavior is motivated by the external incentive of avoiding his legal responsibilities. His urine toxicology screen was negative, thereby eliminating the possibility that he was experiencing a substance-induced mood disorder. Further, there is a clear discrepancy between the patient’s complaint of arm pain and the objective physical examination and radiologic findings. This is characteristic of malingering.

Conversion disorder (choice A) differs from malingering in that the symptom is not intentionally produced and there is no obvious external incentive.

Dissociative identity disorder (choice B) involves the presence of two or more distinct, recurrent identities or personality states. This patient does not show features of this.

Factitious disorder (choice C) involves the intentional production of symptoms, but the motivation is to assume the sick role rather than gain external incentives.

Rheumatoid arthritis (choice E) is a chronic inflammatory disease affecting multiple joints. The patient’s presentation does not align with this condition.

How well did you know this?
1
Not at all
2
3
4
5
Perfectly
6
Q

A 23-year-old man is taking classes at a local community college. He had finished high school without difficulties and was voted “most popular” by his classmates. Three months ago, he started skipping classes after he had a “special revelation” that the “holy spirit” had a more important mission for him. He began worshipping computers, believing them to be sending him special, secret messages from the “holy spirit.” His family, alarmed by the change in his behavior, brings him to the local emergency department. His past history is significant for occasional alcohol and marijuana use. A urine toxicology screen is negative. The patient is admitted to the psychiatric ward of the hospital for further evaluation. In which of the following ways does this person’s presentation preclude a diagnosis of schizophrenia?

(A) Age of onset
(B) Duration of symptoms
(C) Presence of a mood disorder
(D) Severity of symptoms
(E) Type of auditory hallucinations

A

Respuesta: B

The correct answer is B. This patient has schizophreniform disorder. Except for its duration of symptoms, schizophreniform disorder is similar to schizophrenia in diagnostic criteria. Whereas the symptoms must be present for at least 6 months to establish a diagnosis of schizophrenia, an episode of schizophreniform disorder lasts at least 1 month but less than 6 months.

The average age of onset (choice A) for persons with schizophreniform disorder is the same as that for persons with schizophrenia.

One cannot simultaneously have a mood disorder (choice C) in the context of schizophreniform disorder. Only in schizoaffective disorder are symptoms that meet criteria for a mood disorder present.

The symptoms of schizophreniform disorder and schizophrenia may be of equal severity (choice D).

The frequency, character, and description of auditory hallucinations (choice E) may be identical in schizophreniform disorder and schizophrenia.

How well did you know this?
1
Not at all
2
3
4
5
Perfectly
7
Q

A 37-year-old man has a 2-year history of major depression that is managed with sertraline. Several attempts at discontinuing therapy resulted in relapses with major depressive symptoms and strong suicidal ideation. The man visits his psychiatrist to discuss management options. In particular, he discusses problems with his marriage because of sexual dysfunction. He describes a lack of libido that started when he began antidepressant therapy, and he believes that it is destroying his marriage. He discloses the presence of nocturnal erections but a lack of interest in sexual intimacy with his wife. Their relationship is otherwise well, and he doesn’t know what he should do. Which of the following medications is an appropriate antidepressant alternative for relieving his symptoms of sexual dysfunction?

(A) Amitriptyline
(B) Bupropion
(C) Fluoxetine
(D) Paroxetine
(E) Sildenafil

A

Respuesta: B

The correct answer is B. Antidepressant-induced sexual dysfunction is a common condition and can be a major obstacle to long-term therapy for major depression. Bupropion, a reuptake inhibitor of norepinephrine and dopamine, is an appropriate alternative to the major classes of antidepressants because it has a low-to-zero risk for sexual dysfunction.

Amitriptyline (choice A) is unlikely to alleviate the symptoms due to a similarly high risk for sexual side effects.

Fluoxetine (choice C) and paroxetine (choice D) are both SSRIs and are unlikely to change the symptoms of sexual dysfunction.

Sildenafil (choice E) may help reverse sexual dysfunction from antidepressant use in both men and women but does not address the patient’s ongoing need for antidepressant therapy.

How well did you know this?
1
Not at all
2
3
4
5
Perfectly
8
Q

A 28-year-old, malodorous woman in dirty clothing is brought by police into a psychiatric community crisis response center after being found shouting in the middle of a busy street. The woman reportedly was yelling about the end of the world coming soon and that all cars must be abandoned to save the earth from eternal destruction. She screamed at the police who brought her into custody, calling them “evil.” She visibly shook when taken by police car, crying out that “cars are the means to destruction.” Further, she was noted to stare off into the distance and talk to herself during the car ride. On her arrival at the crisis center, her behavior quickly escalated. She became belligerent, shouting obscenities and trying to hit everyone around her. To calm her down and in an attempt to speak with her in a less agitated fashion, the psychiatric intern on call ordered an intramuscular medication to be administered. Which of the following medications is the best choice for rapid tranquilization in this agitated patient?

(A) Clozapine
(B) Haloperidol
(C) Lithium
(D) Olanzapine
(E) Perphenazine

A

Respuesta:

The correct answer is B. This patient is clearly psychotic and shows evidence of a formal thought disorder. Her acute presentation in the crisis center demonstrates the agitation that often accompanies acute psychosis. Often, after calming a patient down with medication and perhaps allowing her to sleep for a short while, the patient is much more calm, cooperative, and redirectable. This not only enables the psychiatric intern to have an easier time with the interview, diagnosis, and immediate treatment planning, but also provides the most safety for the patient. Haloperidol is a typical high-potency antipsychotic: it has a quick onset of action, is reliably administered in all of its forms (i.e., PO, IM, and IV), and can temper the agitation of acute psychosis. It is not unusual for haloperidol to be given in conjunction with IM lorazepam in severe acute agitation.

Clozapine (Choice A) is not used in acutely agitated patients, and is associated with more frequent seizures unless the initial daily dose is minimal. This antipsychotic is very effective in patients who have treatment-resistant psychotic disorders, most often schizophrenia. It has been used to treat aggression in some patients with a broad variety of neuropsychiatric disorders, including traumatic brain injury and mental retardation, but is not used in acute, agitated psychosis as in this scenario.

Lithium (Choice C), an antimanic medication, is the most widely used treatment for the bipolar disorders. It has proved useful in the treatment of acute episodes of mania and depression and, perhaps most significantly, in the long-term prophylaxis of the bipolar disorders. It is not effective acutely.

Olanzapine (Choice D) is a newer, atypical antipsychotic that is not used in acute psychosis because of its delayed onset of action compared with haloperidol. This is mostly because olanzapine is not yet available in an IM formulation, and it undergoes first-pass metabolism before being systemically absorbed. Olanzapine is as effective as haloperidol in the treatment of both first-episode and chronic schizophrenia, but not for an acute presentation.

Perphenazine (Choice E) is a medium-potency, typical antipsychotic that is not used in the acute treatment of agitated psychosis. Like olanzapine, it is not available in IM form. Likewise, it is effective in first-episode and chronic schizophrenia. However, because of several undesirable side effects, it is not currently considered a first-line drug in the treatment of schizophrenia, including acutely agitated psychotic patients.

How well did you know this?
1
Not at all
2
3
4
5
Perfectly
9
Q

An emergency department physician orders a 5-mg haloperidol injection for a psychotic 30-year-old man. Six hours later, the patient’s temperature is 39.4 C (103.0 F), and there is diffuse muscular rigidity and diaphoresis. Over the next 24 hours, the patient becomes increasingly obtunded. He is put on a cardiac monitor, and his blood pressure is noted to fluctuate from 100/70 mm Hg to 170/96 mm Hg. Which of the following is the most likely diagnosis?

(A) Catatonia
(B) Malignant hyperthermia
(C) Neuroleptic malignant syndrome
(D) Serotonin syndrome
(E) Tardive dyskinesia

A

Respuesta: C

The correct answer is C. Neuroleptic malignant syndrome is a life-threatening adverse reaction to antipsychotic medications. It typically develops early in the course of treatment, although it can occur at any point, including after prolonged treatment. It is believed to be caused by impairment of CNS dopamine systems, either from dopamine receptor blockade or following treatment with a presynaptic dopamine depleting-agent like tetrabenazine. In addition to fever, muscle rigidity, and mental status changes, patients may develop autonomic instability, tremor, or dystonia. The most common laboratory abnormality is an elevated creatine kinase. Treatment involves discontinuation of the drug and supportive care, usually in an intensive care unit. Dopamine agonists, such as bromocriptine, are often given in conjunction with dantrolene, which acts as a muscle relaxant. Mortality can be as high as 25% without specific treatment. Symptoms resolve within 10 days.

Catatonia (Choice A) is characterized by mutism and stupor, and may be present in patients with schizophrenia or other psychotic disorders. It may also include bizarre motor behavior, such as sustained postures and stereotypies. Catatonia may respond acutely to treatment with benzodiazepines or electroconvulsive therapy.

Malignant hyperthermia (Choice B) follows administration of general anesthetic agents, not neuroleptics. It is believed to be caused by excessive release of calcium by the sarcoplasmic reticulum, resulting in severe, sustained muscle contraction. Dantrolene is also used to treat this disorder.

Serotonin syndrome (Choice D) is a potentially lethal reaction due to excess serotonin activity associated with combined use of a selective serotonin reuptake inhibitor (SSRI) and an MAO inhibitor or overdosage with an SSRI. It shares certain features of the neuroleptic malignant syndrome, including confusion and diaphoresis. Patients may also be hypertensive and tachycardic.

Tardive dyskinesia (Choice E) presents as a movement disorder developing many months to years after chronic antipsychotic use. It would not develop acutely and would not have any effect on temperature or mental status. Involuntary movements classically involve the oro-buccal-lingual musculature in this condition.

How well did you know this?
1
Not at all
2
3
4
5
Perfectly
10
Q

A 22-year-old college student presents to her physician complaining of increasing apathy and lethargy over the past several weeks. She has also been eating and sleeping more, and her grades have begun to drop. She also states that some of her classmates find her a bit overbearing and irritating, particularly when she goes for days at a time without sleeping while participating in her many student activities. She states that her alcohol intake is limited to two or three beers on occasional weekends while socializing with friends, and she denies any other substance use. Around 6 months ago, she spent almost $1000 on clothing and other gifts for her boyfriend, and she spent another large amount of money on a computer to use to write her latest novel. She states that she was put on paroxetine as a teenager and she “didn’t sleep for days.” Which of the following is the most appropriate single agent to use for pharmacotherapy in this case?

(A) Alprazolam
(B) Amitriptyline
(C) Bupropion
(D) Fluoxetine
(E) Lithium carbonate

A

Respuesta: E

The correct answer is E. The most likely diagnosis is bipolar disorder, most recent episode depressed. The patient’s recent symptoms suggest that she previously had an episode of mania, and this is supported by her description of disinhibition when started on a traditional selective serotonin reuptake inhibitor (SSRI) as a teenager—an event that is sometimes reported when patients with bipolar disorder are started on these agents. Lithium carbonate is more effective in treating bipolar disorder than is an SSRI, particularly in dealing with the cycles of depression and mania. Her hypersomnia and atypical symptoms of depression, combined with her gender, younger age of onset, and history of a manic episode, point to the depression of bipolar disorder rather than major depressive disorder. Therefore, lithium would be the most effective treatment.

Alprazolam (Choice A) is a short-acting benzodiazepine, which is not used to treat depression.

Amitriptyline (Choice B) is a tricyclic antidepressant, which would not treat the mood instability that the patient describes previously having.

Bupropion (Choice C) is a dopamine-norepinephrine reuptake inhibitor used to treat major depressive disorder, not as a single agent in the depression of bipolar disorder.

Fluoxetine (Choice D) is an SSRI similar to paroxetine used to treat depression, and would not be appropriate as a single agent in this case to treat a patient with a history of mania.

How well did you know this?
1
Not at all
2
3
4
5
Perfectly
11
Q

Parents of a 3-year-old boy take their child to the pediatrician for evaluation of what they consider to be “abnormal” behavior. They describe their son as never wanting to be held, even as a baby, not making good eye contact with anyone, crying whenever he is bathed, and having no interest in playing with other children. He does not have any favorite toys, security play items, or any known make-believe friends. He has minimal ability to speak in a coherent language, although the parents believe that they can understand him when he is trying to communicate. The pediatrician observes the boy in the office playroom. He is sitting alone in the corner of the playroom, carefully piling blocks one on top of the other. He is noted to make occasional circular gestures with his left hand. There are four other children, aged 2-4 years, waiting to be seen by the pediatrician; these children are playing together, apart from the boy. In which of the following areas is this boy most likely to have difficulty?

(A) Attention
(B) Concentration
(C) Intelligence
(D) Interpersonal relations
(E) Urinary incontinence

A

Respuesta: D

The correct answer is D. This boy likely has autistic disorder, which affects 3-5 per 10,000 persons. The male-to-female ratio is 3:1. This boy demonstrates a qualitative impairment in social interaction as manifested by nonverbal behaviors, such as poor eye contact and lack of desire to be touched. He has failed to develop peer relationships appropriate to his developmental level and does not seek enjoyment or shared pleasure with anyone. He lacks emotional reciprocity to his parents, has a marked delay in the development of spoken language, and demonstrates a stereotyped and repetitive mannerism. These are all attributes of autism. Also, since the boy’s behavior was described as chronic, his impairments cannot be better accounted for by childhood disintegrative disorder. Children with autistic disorder do not know how to regulate social interaction. They often are not interested in others, cannot accurately interpret the facial expressions and body postures of others, and do not understand social reciprocity.

Difficulty in attention (Choice A) is not prototypical for children with autistic disorder. Their attention span is variable and more often is affected by intelligence level rather than the autistic disorder itself.

Inability to concentrate (Choice B) is also not a characteristic marker of children with autistic disorder. Such children are often preoccupied with aspects of their inner world and with one or more stereotyped and restricted patterns of interest. This is often abnormal either in intensity or in focus. They may concentrate more on one area of focus than another, but this is common for normal children as well and is not viewed as a diagnostic component of autistic disorder.

Intelligence (Choice C) is variable in autistic disorder. Children can be high or low functioning depending on several variables, including intelligence, ability to communicate, and severity of repetitive behaviors, as well as other symptoms. Seventy percent of affected children have IQs measured below 70, and 50% have IQs below 50. Autistic disorder is considered to have an organic basis with no specific site of organic damage. The autistic character portrayed by Dustin Hoffman in the movie “Rainman,” an idiot savant with talents at the genius level, is very rare.

Urinary incontinence (Choice E) is no more prevalent in children with autistic disorder than in normal children.

How well did you know this?
1
Not at all
2
3
4
5
Perfectly
12
Q

A 15-year-old boy is brought in by his mother to see a psychiatrist for “strange behavior.” She reports that her son is often late for school because he spends more than an hour in the shower every morning. When asked about this, he says that he takes a long time because he feels compelled to wash himself in a certain manner, and has to repeat the whole process if he makes a mistake. He knows it sounds ridiculous, and that it makes him late for school and other activities, but he cannot seem to stop himself. In addition, he has found it difficult to fall asleep at night because he needs to constantly check that he has set his alarm for the morning. Which of the following is the most likely diagnosis?

(A) Attention deficit/hyperactivity disorder
(B) Bipolar disorder
(C) Generalized anxiety disorder
(D) Major depressive disorder
(E) Obsessive-compulsive disorder

A

Respuesta: E

The correct answer is E. In obsessive-compulsive disorder (OCD), patients may either experience intrusive thoughts (obsessions) or perform ritualized activities (compulsions), or do both. Obsessions with cleanliness and orderliness, as illustrated by the patient’s showering ritual, are common in OCD. Obsessions and compulsions (such as clock checking) can be so severe that they impact on functions of daily living.

Although OCD and attention deficit/hyperactivity disorder (ADHD; Choice A) may co-exist, this vignette does not demonstrate ADHD, which is characterized by an inability to maintain focus, easy distractibility, and difficulty with behavioral control.

Bipolar disorder (Choice B) is a mood disorder characterized by sustained extremes of mood states (depression and mania). Obsessions and compulsions are not features of bipolar disorder.

Although OCD is categorized as an anxiety disorder, it is distinct from generalized anxiety disorder (Choice C), which is characterized by excessive worry about several different events and activities. These patients may worry about things that elicit anxiety, but they do not experience obsessions or perform compulsions.

Patients with major depressive disorder (Choice D) have either depressed mood or anhedonia (loss of interest) and a constellation of other symptoms, including change in appetite, disturbance in sleep, loss of energy, and decreased concentration. Many patients with OCD develop major depressive disorder, and pharmacotherapy is the same in most cases; however, the case illustrated above is not consistent with major depression.

How well did you know this?
1
Not at all
2
3
4
5
Perfectly
13
Q

A 75-year-old woman experiences several episodes of syncope. She is evaluated and found to have critical aortic stenosis. She is otherwise healthy and, with the advice of her cardiologist, decides to undergo valve replacement surgery. Three days after the surgery the patient is found to be irritable, have a labile mood, and be awake for much of the night. She yells at the nurses at 4 AM one morning for not helping her get into her street clothes; however, the next morning at 8 AM, when the surgical intern came to see her, she was calm and cooperative. She did not remember the events of the previous night. The patient had no known psychiatric history and got a 30/30 on the Folstein Mini-Mental State Examination 1 week prior to the surgery. Which of the following is the most likely diagnosis?

(A) Adjustment disorder
(B) Cyclothymia
(C) Delirium
(D) Dementia
(E) Malingering

A

Respuesta: C

The correct answer is C. This patient is delirious. Delirium is a transient disorder of brain function that results in global cognitive impairment and other behavioral phenomena. It is a common disorder, with its main features being impairment of consciousness, attentional deficit, and a fluctuating course. An estimated 10 to 15% of general medical inpatients are delirious at any given time, and studies show that as many as 30 to 50% of acutely ill geriatric patients become delirious at some point during their hospital stay. Risk factors for delirium include new or worsened medical illness, older age, and baseline cognitive impairments, usually secondary to dementia. The syndrome of delirium is almost always due to an identifiable cause, such as systemic or cerebral disease or drug intoxication or withdrawal.

Adjustment disorder (Choice A) is characterized by the development of emotional or behavioral symptoms, often related to anxiety or depression, in the context of a psychosocial stressor. The symptoms resolve when the stressor ends, or when the patient learns to adapt to them better. Symptoms of severe confusion, as were present in this patient, are unusual in adjustment disorders, as is the sudden resolution of symptoms despite the continuing stressor.

Cyclothymia (Choice B) is a disorder characterized by symptoms that have occurred for at least 2 years. This patient has been suffering for only 3 days. In cyclothymia, there are numerous periods of hypomanic, but not manic, symptoms and of depressive symptoms that do not meet criteria for a major depressive episode. By definition, the person has not been symptom-free for more than 2 months at a time.

Dementia (Choice D) is characterized by the development of multiple cognitive deficits manifested by memory impairment, and by one or more of the following features: aphasia (language disturbance), apraxia (impairment in carrying out motor activities), agnosia (failure to recognize or identify objects), or disturbance in executive functioning. The onset is usually gradual and cannot occur exclusively during the course of a delirium. As stated above, a person with dementia is more susceptible to delirium.

The characteristic feature of malingering (Choice E) is the intentional production of false or markedly exaggerated physical or psychological symptoms as motivated by external incentives. This woman is unlikely to be producing her symptoms intentionally, and there is no obvious external incentive to do so.

How well did you know this?
1
Not at all
2
3
4
5
Perfectly
14
Q

A 52-year-old man is seen for the first time by a psychiatrist. He states that he always feels “keyed up” and “on edge” and has been having nightmares about combat experience from the Vietnam War, from which he is a veteran. He states that he witnessed several of his fellow soldiers get killed in combat and he has since experienced what he describes as “movies of it all happening over again” while awake. He reports that he has difficulty discussing these events because they are so distressing. Prior to Vietnam, he was happily married and had several friends; now he feels that he cannot get close to other people and oftentimes gets angry with people for no apparent reason. Which of the following is the most likely diagnosis?

(A) Acute stress disorder
(B) Generalized anxiety disorder
(C) Posttraumatic stress disorder
(D) Schizoid personality disorder
(E) Sleep terror disorder

A

Respuesta: C

The correct answer is C. The patient describes classic symptoms of posttraumatic stress disorder (PTSD), which is relatively common in patients who have combat exposure. In addition to having witnessed a traumatic event, patients also complain of flashbacks, nightmares, persistent avoidance of stimuli associated with the trauma, and symptoms of increased arousal.

Acute stress disorder (Choice A) is similar to PTSD; however, symptoms are resolved within 4 weeks of the associated event.

In generalized anxiety disorder (Choice B), there is excessive worry about a number of things and there often are somatic manifestations of the anxiety, such as irritability and restlessness. Nightmares, flashbacks, and an identifiable traumatic event are not prominent features.

Schizoid personality disorder (Choice D) is characterized by a pervasive detachment from social relationships. There often is an indifference on the patient’s part with regard to establishing relationships with others. The patient described above is obviously distressed that he can no longer seem to form a connection with others.

Patients with sleep terrors (Choice E) often do not report nightmares. They have the sensation of fear and autonomic hyperactivity on sudden awakening, and there is unresponsiveness to efforts of others to comfort them during the episode.

How well did you know this?
1
Not at all
2
3
4
5
Perfectly
15
Q

A 20-year-old woman with bipolar disorder, who is currently in a manic state, responds to many emergency department interview questions with flight of ideas. The patient is asked how she got to the hospital. Which of the following statements by the patient is the best example of this form of thought disturbance?

(A) “I drove here myself.”
(B) “I was reading a great book at home and, after I finished it, I drove here myself.”
(C) “I really like history books and I was reading one at home today while eating tuna fish.”
(D) “I started driving but was thinking about my history. History books are great. I once wrote a paper on Alexander the Great. I do well in college.”
(E) “The spy in the book I was reading is real, and he was watching me through the pages. He knew who I was.”

A

Respuesta: D

The correct answer is D. Flight of ideas is non-goal directed speech that reflects a rapid jumping of thoughts through a series of weakly related ideas. “I started driving but was thinking about my history. History books are great. I once wrote a paper on Alexander the Great. I do well in college,” demonstrates this process. The sentences are abruptly and tenuously connected. The “great” in “history books” is the topic for the reference to “Alexander the Great,” which was the subject of a paper. The paper was likely an assignment written in college, which led to the reference to “do(ing) well in college.” The patient did not answer the question she was asked.

“I drove here myself” (Choice A) is a goal-directed, linear statement.

“I was reading a great book at home and, after I finished it, I drove here myself” (Choice B) is an example of circumstantiality. The answer is overly detailed, but the question is ultimately answered. The mention of the book is vaguely related to the patient’s driving herself to the emergency department.

“I really like history books and I was reading one at home today while eating tuna fish” (Choice C) is an example of tangentiality. The question is not answered; however, proper words and grammar are used, and ideas flow from a train of thought.

“The spy in the book I was reading is real, and he was watching me through the pages. He knew who I was” (Choice E) is an example of looseness of association. There is a disintegration of a meaningful connection of ideas. In addition, this statement describes paranoia and a delusion of reference and likely also of persecution.

How well did you know this?
1
Not at all
2
3
4
5
Perfectly
16
Q

A 45-year-old man is admitted to the ICU for traumarelated injuries sustained in a car accident. Thirty-six hours after admission he becomes agitated. He is pulling at his IV access lines and is disoriented to place and time. His blood pressure is 190/110 mm Hg, and his pulse is 114/min. A reliable history from the patient’s son reveals that the patient is alcohol dependent. Which of the following is the most appropriate next step in management?

(A) Haloperidol
(B) Lithium
(C) Lorazepam
(D) Propranolol
(E) No medication

A

Respuesta: C

The correct answer is C. This patient is likely beginning to exhibit alcohol withdrawal. Even without full-blown delirium tremens (DTs), alcohol withdrawal can be serious and should be treated. Tremulousness usually develops 6-8 hours after the cessation of drinking. Perceptual symptoms often begin after 8-12 hours, seizures in 12-24 hours, and DTs after 72 hours.
Benzodiazepines are the primary medications for the control of alcohol withdrawal symptoms. Benzodiazepines, such as lorazepam, help with anxiety, shakiness, tachycardia, hypertension, diaphoresis, seizure activity, and delirium. In this situation, lorazepam has advantages over other benzodiazepines in that it can be administered in PO, IM, and IV forms and is not hepatotoxic.

Haloperidol (Choice A) will help with some perceptual disturbances of alcohol withdrawal and may cause a calming effect on agitated behavior, but it will not address the sympathetic hyperactivity, seizures, or alcohol-withdrawal delirium.

Lithium (Choice B) is an antimanic agent used in bipolar disorders and will not treat alcohol withdrawal symptoms.

Propranolol (Choice D) helps block the symptoms of sympathetic hyperactivity; it is not an effective drug for seizures or delirium.

No medication (Choice E) is incorrect because alcohol withdrawal, especially DTs, is a medical emergency that can result in significant morbidity and mortality. Even with treatment, DTs have a mortality rate of approximately 15%. Although this patient is not described as having DTs, treatment with lorazepam will help prevent them from occurring.

How well did you know this?
1
Not at all
2
3
4
5
Perfectly
17
Q

A 29-year-old woman consistently meets “the man of her dreams” only to have the relationships fail after a month or two. She views herself as unworthy of being loved and often hurts herself after these relationships fail. She feels depressed if she has no weekend plans and has difficulty controlling both her anger and sadness if she knows she will be alone. When frustrated, she often acts out, impulsively drinking alcohol and having sex with men whom she does not know well, only to find herself feeling empty afterward. Further, she is unhappy in her job and thinks that she could “do better.” Which of the following is the most characteristic defense mechanism used by people who have this personality disorder?

(A) Displacement
(B) Idealization
(C) Intellectualization
(D) Reaction formation
(E) Splitting

A

Respuesta: E

The correct answer is E. This patient meets the DSM-IV criteria for borderline personality disorder. She has a pervasive pattern of unstable interpersonal relationships, poor self-image, and lack of control of her affect. Her behavior is marked by impulsivity. She feels sad during weekends when she is alone and imagines herself as being abandoned. Her unstable relationships are characterized by alternating between extremes of idealization (“the man of her dreams”) and devaluation.

She appears to have a very poor sense of self, with an inability to find a suitable partner and job. She is impulsive in substance abuse and sexual activity, both of which are potentially self-damaging, and she shows lability of affect because of a marked reactivity of mood. The question did not indicate whether this person is suicidal, although she quite possibly would be; some of the described behaviors of hurting herself may include suicide gestures. One of the most characteristic defense mechanisms associated with borderline personality disorder is splitting, which means that the person psychologically separates positive attributes into one individual and negative attributes into another. Splitting occurs because the person is not able to tolerate her ambivalent feelings toward another individual. Usually, the person is unconscious of her ambivalence toward another and does not realize the extremes of her reaction.

Displacement (Choice A) occurs when the feelings associated with a psychologically unacceptable object, idea, or situation are transferred to another object, idea, or situation. The latter one is often symbolically related to the former. For instance, a woman who is raped by her husband may become angry at the prosecutor who is handling the case because he was 10 minutes late for a meeting.

Idealization (Choice B) occurs when a person unrealistically attributes strictly positive characteristics to another person or situation. In the above example, the person with borderline personality disorder repeatedly believed that she had met the “man of her dreams”; however, this idealization was only short-lived and was quickly replaced with devaluation.

Intellectualization (Choice C) is the transformation of emotionally disturbing events into cognitive challenges that do not recognize the emotional stress. For example, a person who has been rejected for a job she wanted very much says that this is simply a problem of getting her resume into a better format.

Reaction formation (Choice D) occurs when an unacceptable feeling or thought is transformed into its opposite. This often occurs when hate is transformed into love or fear is transformed into empowerment. For instance, a battered woman who has managed to leave her abusive home situation may become involved in work with her local battered women’s shelter.

How well did you know this?
1
Not at all
2
3
4
5
Perfectly
18
Q

A 51-year-old Vietnam veteran who has been treated for several months with antidepressants for posttraumatic stress disorder (PTSD) recently began taking a new sleep medication for insomnia. After several days, he calls his doctor complaining of a prolonged and painful erection. The doctor instructs him to stop the medication and immediately come into the emergency department. Which of the following medications is most likely causing this condition?

(A) Chlordiazepoxide
(B) Hydroxyzine
(C) Mirtazapine
(D) Trazodone
(E) Zolpidem

A

Respuesta: D

The correct answer is D. Priapism is persistent penile erection accompanied by severe pain. It can occur with antidepressants or antipsychotics, but it is most frequently seen with trazodone use (1 in 1000 men). It is a medical emergency that requires evaluation by a urologist.

Chlordiazepoxide (Choice A) belongs to the group of long-acting benzodiazepines and is usually used for detoxification from sedative hypnotics because of its long half-life. It is not used for the treatment of insomnia. Side effects include sedation, amnesia, psychomotor retardation, decreased respiratory response to carbon dioxide, and a potential withdrawal syndrome.

Hydroxyzine (Choice B) is a piperazine derivative that exerts antihistaminic, anticholinergic, mild sedative, and bronchodilator effects. It is used as premedication in anesthesia and in the management of pruritic syndromes. It exerts a mild antianxiety effect. It is not used as a hypnotic. Its side effects include drowsiness and dry mouth.

Mirtazapine (Choice C) is a novel antidepressant. It has sedative properties related to its antihistaminic activity; in lower doses, it can be used for treatment of insomnia. It is mainly indicated for the treatment of depression. Side effects include dizziness, weight gain, and increases in serum lipids and transaminases.

Zolpidem (Choice E) is a nonbenzodiazepine hypnotic used for short-term treatment of insomnia. Its most common side effects include headache, dizziness, drowsiness, gastrointestinal symptoms, generalized pain, and myalgias.

How well did you know this?
1
Not at all
2
3
4
5
Perfectly
19
Q

A 32-year-old man has chronic paranoid schizophrenia and a history of mild tardive dyskinesia. His psychiatrist recently switched his medication to olanzapine taken at bedtime. The patient smokes approximately a pack of cigarettes daily, weighs 70 kg (154 lb) at a height of 5 feet 10 inches (178 cm), and does not drink alcohol. Which of the following would be the most common adverse side effect of olanzapine in this patient?

(A) Gastrointestinal complications
(B) Orthostatic hypotension
(C) Sedation
(D) Tardive dyskinesia
(E) Weight gain

A

Respuesta: C

The correct answer is C. Approximately 30% of patients taking the usual maintenance dose of olanzapine experience sedation, making it the most common side effect of the choices listed. The reason that the medication is given at bedtime is to reduce adverse effects of sedation as much as possible. Other much less common side effects of olanzapine include prolactinemia, dizziness, and akathisia.

Gastrointestinal complications (choice A), particularly constipation, are known to be associated with olanzapine use, but are not as common as sedation.

Orthostatic hypotension (choice B) is also a complication of olanzapine use. Patients who take olanzapine should be cautious when rising in the morning or nighttime hours, or when rising from a sitting position, so that they do not lose consciousness or become unsteady. However, orthostatic hypotension is less common than sedation with olanzapine.

Tardive dyskinesia (choice D), a syndrome of abnormal involuntary movements associated with neuroleptic use, has not yet been reported in patients taking olanzapine.

Weight gain (choice E) is a recognized complication of olanzapine use but is not as common as sedation.

How well did you know this?
1
Not at all
2
3
4
5
Perfectly
20
Q

A 34-year-old man has had a significantly increased appetite, gained 10 lb, and required increased sleep over the past several months. He has felt severely fatigued during the same time period and describes a heavy feeling in his arms and legs. His brother, a psychiatrist, visits him, notes the complaints, and feels that the sensation he described in his arms and legs is pathognomonic for “leaden paralysis.” This is a characteristic feature of which of the following diagnoses?

(A) Conversion disorder
(B) Major depression with atypical features
(C) Schizophrenia, catatonic type
(D) Schizotypal personality disorder
(E) Somatoform disorder

A

Respuesta: B

The correct answer is B. Major depression with atypical features is characterized by mood reactivity, in which the patient’s mood brightens in response to actual or potentially positive events, and two or more of the following features: leaden paralysis, significant weight gain or increase in appetite, hypersomnia, or a long-standing pattern of interpersonal rejection sensitivity that results in significant social or occupational impairment. This type of depression differs from other types in that its characteristic psychomotor disturbances are opposite those of others. In other depressions, there is a lack of reactivity to usually pleasurable stimuli. Affected patients do not feel much better, even temporarily, when something good happens. Likewise, in other types of depression, there is usually anorexia and weight loss, and early morning awakening commonly marks the sleep pattern. Further, there typically are no complaints of either a heavy feeling in one’s arms or legs or a sensitivity-based concern of rejection. Atypical depression responds especially well to MAOIs.

Conversion disorder (choice A) is a disturbance of bodily functioning that does not conform to the current concepts of the anatomy and physiology of the central or peripheral nervous systems. It is characterized by one or more symptoms affecting voluntary motor or sensory function that suggest a neurologic or medical condition but cannot be explained by one. It typically occurs in a setting of stress and tends to transform the psychic energy of the turmoil of acute conflict into a personally meaningful metaphor of bodily dysfunction. The symptom is not intentionally produced, as in a factitious disorder or malingering.

Schizophrenia, catatonic type (choice C), is marked by at least two of the following features: immobility suggestive of either catalepsy or stupor, excessive and apparently purposeless motor activity, extreme negativism, peculiar voluntary movement such as posturing, stereotypy, mannerisms, grimacing, echolalia, or echopraxia. There are no common somatic complaints, and leaden paralysis is not typical of this disorder.

Schizotypal personality disorder (choice D) is in the cluster A category of personality disorders. It is marked by a pervasive pattern of social and interpersonal deficits characterized by acute discomfort with, and reduced capacity for, close relationships. There also are cognitive or perceptual distortions and eccentricities of behavior that begin by early adulthood. Furthermore, there must be five of the following features: ideas of reference, odd beliefs that influence behavior, unusual perceptual experiences, odd thinking and speech, suspiciousness, inappropriate affect, eccentric behavior, and lack of close friends. As in schizophrenia, there are no common somatic complaints, and leaden paralysis is not typical of this disorder.

Somatoform disorder (choice E) is not a disorder in and of itself. There are five specific somatoform disorders: somatization disorder, conversion disorder, pain disorder, hypochondriasis, and body dysmorphic disorder.

How well did you know this?
1
Not at all
2
3
4
5
Perfectly
21
Q

Available clinical evidence indicates that individuals who are homozygous for the Apo E4 gene are at increased risk of developing Alzheimer disease. However, laboratory testing aimed at the identification of Apo E4 carriers does not appear adequate as a screening test for early detection of this risk factor. Which of the following is the most commonly suggested reason for not using Apo E allele determination as a screening test?

(A) The disease does not have a preclinical (asymptomatic) stage
(B) The disease is not preventable or treatable
(C) The disease is not sufficiently prevalent
(D) The disease is not sufficiently serious
(E) The test is not sufficiently accurate
(F) The test is not sufficiently sensitive

A

Respuesta: B

The correct answer is B. There are three allelic forms of this gene, e2, e3, and e4. Clinical studies have shown that the allele e4 of apolipoprotein E (Apo E) increases the risk for late-onset Alzheimer disease (AD), especially in homozygous individuals. However, its use as a screening tool to identify individuals who have an increased risk of developing AD is highly controversial, mainly because there are no known effective methods to prevent AD or cure it once it manifests. The clinical usefulness of a screening test for early detection of diseases or predisposing factors is based on a number of criteria, including the following: the disease must be sufficiently serious, prevalent, treatable, or preventable, and the test should be sufficiently accurate (i.e., sensitive and specific). Furthermore, a presymptomatic stage should precede the onset of the disease to allow preclinical or early detection.

AD has a long preclinical stage (compare with choice A) before its earliest manifestations.

The prevalence of AD is very high (compare with choice C). Approximately 6% of persons older than 65 have varying degrees of AD.

AD is a severe form of dementing disorder and ultimately leads to death within an average of 7-10 years following clinical onset (compare with choice D).

Testing for Apo E phenotype is both sensitive (compare with choice F) and specific (compare with choice E).

22
Q

A teacher calls the parents of an 8-year-old boy to discuss his poor adjustment to the second grade. He has difficulty sitting still for group activities and needs constant reminders from the teacher not to hit his classmates. He has problems listening in class and does not complete assignments without several reminders. These multiple reminders and behavioral reprimands do not seem to be leading to any improvement. His parents have also been noticing similar problems at home. The boy is sent to a child psychiatrist, who notes increased psychomotor activity with restlessness during the evaluation. The testing revealed no learning disability, and the boy was diagnosed with attention-deficit hyperactivity disorder. This boy is at a higher risk than the normal population to have which of the following comorbid disorders?

(A) Asperger disorder
(B) Autistic disorder
(C) Anxiety disorder
(D) Oppositional defiant disorder
(E) Schizophrenia, undifferentiated type

A

Respuesta: D

The correct answer is D. The most common comorbid disorders found in both clinical and epidemiologic samples of children with attention-deficit hyperactivity disorder (ADHD) are oppositional defiant disorder (ODD) and conduct disorder (CD). Typically these children display argumentative behavior and attitudes, temper tantrums, defiance of authority and rules, and aggressive, antisocial behavior in addition to the symptoms of ADHD. The rate of concurrent ADHD and ODD is 35%; the combination of concurrent ODD or CD and ADHD is 50 to 60%. Interestingly, school-aged children with ODD or CD almost invariably meet criteria for ADHD; yet, it is more common for adolescents with ODD or CD to not have a concurrent diagnosis of ADHD. Other influences that seem to correlate ODD and CD with ADHD include greater symptom severity, reading disorder, lower socioeconomic status, and parental alcoholism. Short-acting psychostimulants are the first- line treatment for the pharmacotherapy of ADHD, mostly because of their ability to improve both behavioral and cognitive problems in 70 to 80% of affected children. However, behavioral improvements do not always lead to complete remission of symptoms. There often is significant residual ADHD symptomatology, as well as peer and academic problems, even after treatment.

Asperger disorder (choice A) is a pervasive developmental disorder characterized by qualitative impairment in social interaction, restricted and repetitive stereotyped patterns of behavior, and no clinically significant language or cognitive delay. Persons with this disorder do not have difficulty in the development of age- appropriate self-help skills or adaptive behavior that children with ADHD often have. Rather, children with Asperger disorder often do not play with others and are seen more as aloof, introverted, and bizarre in comparison to the disruptive and extroverted behavior of children with ADHD.

Autistic disorder (choice B), like Asperger disorder, is a pervasive developmental disorder. It causes impairment in social interaction, often with the use of multiple nonverbal behaviors, failure to develop peer relationships, and lack of social or emotional reciprocity. There is a problem in communication, often with delay or total lack in the development of spoken language and stereotyped behavior that may encompass an inflexible adherence to a preoccupation with a specific, nonfunctional routine. Autistic persons often have low IQs and cannot tolerate standard classroom environments. They do not present similarly to children with ADHD.

Anxiety disorder (choice C) is incorrect. Nonetheless, significant comorbidity exists with ADHD and anxiety disorders. The average comorbidity is 25%. With the early onset of ADHD, most diagnoses of anxiety disorders are made after the emergence of ADHD. This suggests that some instances of ADHD-anxiety disorder comorbidity are possibly secondary to the experience of enduring the chronic disorder of ADHD itself. Some differences in patterns of performance on cognitive tasks and reduced cognitive improvement with stimulants distinguish children with ADHD plus anxiety and those without anxiety.

Schizophrenia, undifferentiated type (choice E) is incorrect. There is no known correlation between ADHD and schizophrenia of any type.

23
Q

A 22-year-old female student presents to her physician after collapsing in the cafeteria following loud joking and laughter with her friends. She describes loss of muscle tone after she fell but no loss of consciousness. On further examination, she reveals a 4- month history of increased daytime sleepiness, with several episodes of falling asleep during her classes. She believed that the episodes were related to poor sleep because of vivid dreams she had on falling asleep. Which of the following is the most likely diagnosis?

(A) Catalepsy
(B) Kleine-Levin syndrome
(C) Narcolepsy
(D) Periodic paralysis
(E) Primary hypersomnia

A

Respuesta: C

The correct answer is C. Narcolepsy is diagnosed by irresistible attacks of refreshing sleep that occur during the day over at least 3 months. It is characterized by the presence of cataplexy (brief episodes of sudden loss of muscle tone throughout the body mostly in association with intense emotion) or recurrent intrusions of REM sleep in transition between sleep and wakefulness, as manifested by hypnagogic or hypnopompic hallucinations. Narcolepsy occurs most frequently before the age of 30. Sleep paralysis, characterized by the inability to move in the presence of preserved consciousness, is another typical symptom on waking up.

Catalepsy (choice A) is a term for motoric immobility maintained voluntarily and is most commonly seen as a part of schizophrenic symptomatology.

Kleine-Levin syndrome (choice B) is a rare, self-limiting condition consisting of recurrent episodes of prolonged sleep lasting one to several weeks. It begins in childhood, and the wakeful periods are marked by social withdrawal, irritability, confusion, and frank psychotic symptoms, such as delusions or hallucinations.

Periodic paralysis (choice D) is a group of hereditary muscle disorders inherited in autosomal-dominant fashion; it develops in early childhood and adolescence. Attacks occur at rest following heavy exercise or meals rich in carbohydrates. During the attacks, patients are unable to move their limbs. Serum levels of potassium can be below, at, or above normal limits, depending on the type of disorder. Treatment is directed at controlling serum potassium and preventing attacks.

Primary hypersomnia (choice E) is a syndrome of excessive sleepiness for at least 1 month, as evidenced by prolonged sleep or daytime sleep episodes. It causes clinically significant impairment in social functioning. It does not occur secondary to the effects of substance abuse or a general medical condition and does not happen during the course of another sleep disorder.

24
Q

A 27-year-old woman, after undergoing elective cosmetic breast augmentation surgery, presents to her surgeon dissatisfied with the results and requesting more surgery, even though she has received compliments from both her friends and her boyfriend concerning her new appearance. She has had no medical complications from the procedure. The patient, who weighs about 110 pounds, tells her surgeon that she has been preoccupied with her breast size since she was a teenager and that she has had difficulties in relationships with previous boyfriends because she felt that they viewed her as inadequately feminine. She describes that she has also had difficulties dating in the past because of the excessive amount of time she spends working out at the gym and jogging to maintain her figure. Which of the following would be the most appropriate diagnosis in this patient?

(A) Adjustment disorder with disturbance of emotion
(B) Body dysmorphic disorder
(C) Borderline personality disorder
(D) Hypochondriasis
(E) Somatoform disorder, not otherwise specified

A

Respuesta: B

The correct answer is B. Body dysmorphic disorder involves a preoccupation with an imagined defect in appearance, which causes clinically significant distress or impairment in social, occupational, or other important areas of functioning. The preoccupation is not better accounted for by another mental disorder, such as the dissatisfaction with body shape and size found in anorexia nervosa. The fact that this patient has had social difficulties because of her perception of her body shape points to body dysmorphic disorder as the most appropriate diagnosis.

Adjustment disorder with disturbance of emotion (Choice A) requires both a recent stressor and some symptoms of depression, which this patient does not report.

The patient does not have evidence of borderline personality disorder (Choice C), as she does not have an unstable self-image or sense of self, and she does not have any recurrent suicidal behavior or gestures required for this diagnosis.

Hypochondriasis (Choice D) requires a preoccupation with fears of having a serious disease based on the person’s misinterpretation of bodily symptoms. The belief is not restricted to a circumscribed concern about appearance (as in body dysmorphic disorder).

Somatoform disorder, not otherwise specified (Choice E), includes false beliefs (for example, that one is pregnant when actually not) or disorders that involve unexplained physical complaints (such as fatigue or body weakness) of less than 6 months’ duration that are not due to another mental disorder.

25
A concerned mother asks a child psychiatrist whether her child’s behavior is abnormal. She describes the child as typically happy: she smiles often at her parents, is a good eater, and is mostly a good sleeper, able to sleep through the night without waking. However, when the parents leave her with a baby-sitter on occasional weekend nights, the child reportedly screams uncontrollably. The baby-sitter finds it difficult to feed her, and likewise she rarely falls asleep without a hysterical crying outburst. Which of the following is the most likely age of this child? (A) 1 week (B) 3 months (C) 8 months (D) 18 months (E) 24 months
Respuesta: C The **correct answer is C**. A number of child psychiatrists have written about various theories of child development. Multiple models exist that concern the organization of cognition, attachment, learning, aggression, social skill development, and individuation. The child in this question is in a normal stage of development and is experiencing stranger anxiety. A fear of strangers is usually first noted in infants at about 26 weeks of age, but does not develop fully until about the age of 32 weeks, or 8 months. When a stranger approaches an 8-month-old infant, he will often cry and cling to his mother. Infants exposed to many caretakers, most often in an extended family or a daycare setting, are less likely to have stranger anxiety than those infants exposed to only one caretaker. Stranger anxiety is believed to be a result of an infant’s increasing ability to distinguish caretakers from all other persons. Separation anxiety, which usually occurs between 10 and 18 months of age, is related but not identical to stranger anxiety. Separation from the person to whom the infant is attached instigates separation anxiety, whereas stranger anxiety occurs even when the infant is in his mother’s arms. In separation anxiety, an infant learns to crawl or walk away from his mother but often looks back at her to reassure himself that she is still there.
26
A 35-year-old woman, who was hospitalized 6 months ago and diagnosed with bipolar disorder, mixed type, arrives for a follow-up appointment. She feels that she is stable on her medication but has noticed an increased appetite with significant weight gain, as well as hair loss, since her discharge. Her physical examination and laboratory tests, including a thyroid panel, are unremarkable. Her physician concludes that her medication is the cause of her problems. Which of the following medications is this patient most likely taking? (A) Clonazepam (B) Divalproex (C) Gabapentin (D) Lithium (E) Olanzapine
Respuesta: B The **correct answer is B**. Divalproex, a preparation of valproic acid, is an anticonvulsant medication that has also been approved by the FDA for treatment of manic episodes. Its common side effects include nausea, sedation, weight gain, and transitory hair loss, as well as a transitory increase in liver function tests. Rare side effects include hepatitis and pancreatitis, as well as a possible decrease in platelets or platelet dysfunction. Clonazepam **(Choice A)** is a benzodiazepine that is used in place of or in conjunction with an antipsychotic to treat acute manic agitation. It is used for short-term treatment of anxiety, insomnia, or catatonia associated with mania or depression. It can also be used as adjunctive maintenance treatment along with mood stabilizers, but lacks specific antimanic, antidepressant, or mood stabilizing properties. Clonazepam is also used as an anticonvulsant. Side effects include drowsiness and ataxia. Gabapentin **(Choice C)** is a new antiseizure medication that has been approved as adjunctive therapy for treatment of refractory partial seizures. In some cases, it has been used as adjunctive medication for treatment of refractory bipolar disorder. Its mood stabilizing properties have not been confirmed. The most common side effects include somnolence, dizziness, and ataxia. Lithium **(Choice D)** is the treatment of choice for bipolar I disorder. It is less effective in mixed bipolar disorder. Symptoms of lithium toxicity include diarrhea, severe tremor, polyuria, ataxia, confusion, seizures, gastric distress, weight gain, tremor, fatigue, and cognitive deficits. Other side effects involve the kidneys, heart, thyroid gland, and skin. Olanzapine **(Choice E)** is an atypical antipsychotic that also has some mood stabilizing properties. It has also been used recently in the treatment of bipolar disorders. Its most common side effects include postural hypotension, constipation, weight gain, and dizziness. However, hair loss is not one of the side effects of this medication.
27
A breast cancer patient starts to feel nauseated as she exits the hospital corridor leading to the chemotherapy treatment area and enters the waiting room. After she opens the waiting room door, she feels even more nauseated. As she waits in line to check herself in for her eighth of ten chemotherapy treatments, she vomits. This reaction is an example of which of the following learning theories? (A) Classical conditioning (B) Covert sensitization (C) Habituation (D) Learned helplessness (E) Operant conditioning
Respuesta: A The **correct answer is A**. Classical conditioning is learning that takes place when two events occur closely together in time. According to this theory, learning occurs when an initially neutral stimulus, the conditioned stimulus, is paired with a stimulus that naturally elicits a response, the unconditioned stimulus (chemotherapy). In this example, the conditioned stimulus is the entrance into the waiting room. The response (nausea and vomiting) elicited by the unconditioned stimulus is called the unconditioned response. After repeated and contiguous pairing of the two stimuli (in this instance, eight pairings of the waiting room and nausea and vomiting), the conditioned stimulus elicits the unconditioned response, which then is called the conditioned response. The learning theory of classical conditioning has been influential in the conceptualization of certain clinical disorders, as well as in developing some treatment methods. It is particularly useful in understanding and treating phobic disorders, appreciating stress responses that were not part of one’s initial trauma, and cueing exposure treatment for alcoholics and drug addicts. Covert sensitization **(Choice B)** is a method of reducing the frequency of behavior by associating it with the imagination of unpleasant consequences. This is a treatment method used in circumstances during which a patient wants to decrease the amount of an unwanted behavior. Habituation **(Choice C)** is a simple form of learning in which the response to a repeated stimulus lessens over time. Learned helplessness **(Choice D)** describes the cognitive experience of a person who perceives unpredictable and uncontrollable events as the basis for feeling helpless and powerless. Operant conditioning **(Choice E)** is a form of learning in which behavioral frequency is altered through the application of positive and negative consequences.
28
A 6-year-old boy learns how to paint with watercolors in his first- grade art class. He likes to learn new skills and works hard at painting a picture of himself outside his house. Despite gross irregularities in the sizes and shapes of the objects in the picture, he is very proud of himself and feels competent. He brings the picture home from school and announces to his mother, “Mommy, look at the beautiful picture I made!” According to Erik Erikson, this child is at which of the following stages of development? (A) Autonomy vs. shame and doubt (B) Identity vs. role confusion (C) Industry vs. inferiority (D) Initiative vs. guilt (E) Trust vs. mistrust
Respuesta: C The **correct answer is C**. Erikson conceptualized eight stages of ego development across the life cycle. Each stage represents a point along a continuum of development in which physical, cognitive, instinctual, and sexual changes combine to create an internal crisis whose resolution results in either psychosocial regression or growth. The boy in this example is in Erikson’s psychosocial stage of industry vs. inferiority. This stage occurs at approximately 5 to 13 years, during which time the child discovers the pleasures of productivity and the feelings of competency. He develops industry by learning new skills and taking pride in things made. If the child is not ready for this stage, either by lack of resolution of a previous stage or by a problem that he is experiencing, the child may develop a sense of inferiority and inadequacy. Autonomy vs. shame and doubt **(choice A)** occurs from about age 18 months to 3 years. While the toddler is developing speech and sphincter control, he practices holding on and letting go and, according to Erikson, experiences the beginning of his will. At this stage, a toddler is learning to develop his own internal controls and is learning the consequences of his lack of self-control or judgment. He is learning to have self-certainty as opposed to self- consciousness and, if successful in this stage, will learn not to find shame in and doubt himself. Identity vs. role confusion **(choice B)** occurs from about 13 to 21 years. During the phase of puberty and its associated physiologic and social changes, the adolescent becomes focused on the question of identity. The formation of cliques and the intolerance of individual differences are methods of dispelling a sense of identity loss. According to Erikson, during this psychosocial stage, a person’s goal is to develop a more sharply focused identity and develop a sense of faithfulness to one’s idea of self. Role confusion occurs when the adolescent is unable to attain a sense of self and belonging. Initiative vs. guilt **(choice D)** occurs from about 3 to 5 years of age. In this stage, Erikson describes the child as increasing his mastery over locomotor and language skills, stimulating curiosities of the outside world. The child is naturally active and intrusive and, if successful at this stage, learns the basis for the subsequent development of realistic ambition and the purpose of virtue. If the child is unsuccessful, however, he has overwhelming guilt over the drive for conquest and anxiety over anticipated punishment. Trust vs. mistrust **(choice E)** is the first of Erikson’s stages, occurring from birth to approximately 18 months. The task of the infant is to trust his surroundings. This occurs most easily if there is a mother available who is able to anticipate and respond to her baby’s needs in a consistent and timely manner. If the baby does not acquire a sense of trust and pleasantness in his world, he will feel hopeless and will tend to isolate himself and be distrustful of others.
29
A 17-year-old girl is caught stealing three lipsticks from a local convenience store. The security guard takes her to the back of the store and gathers information from her. The girl hands over her driver’s license, and the guard calls the police with her identifying information. He learns that this girl has been caught stealing several times before, with reports from convenience, clothing, and food stores. The police come to the store and arrest her. Which of the following attributes is likely to be absent in this individual? (A) Delusional thinking (B) Displeasure at the thought of being caught in the act of the theft (C) Failure to resist impulses to steal (D) Increasing tension before the theft (E) Pleasure at the time of committing the theft
Respuesta: A The **correct answer is A**. The most likely disorder that might be responsible for this girl’s behavior is kleptomania, an impulse-control disorder. Kleptomania is characterized by a recurrent impulse to steal objects that are not needed for personal use or monetary value. According to the DSM-IV, the stealing is not due to anger or revenge, is not associated with a delusion or hallucination, and is not due to conduct disorder, a manic episode, or antisocial personality disorder. Pathologic stealing is estimated to occur in 6 per 1000 persons. Fewer than 5% of shoplifters actually have kleptomania. There seems to be a preponderance of female kleptomaniacs; however, these data may be skewed since, in general, women are more likely than men to present for psychiatric evaluation. Similarly, women are more likely to be referred for psychiatric evaluation by the courts, as opposed to men who are more often sent to jail. At first presentation, the average age for women is 35; for men, 50. A high rate of pathologic stealing occurs in bulimia nervosa, and, given that kleptomania is at least somewhat responsive to serotonergic agents, these conditions may actually have a common pathophysiologic connection. Delusional thinking is absent because among the criteria listed in the DSM-IV is that the person cannot be responding to a delusion or hallucination when stealing. Persons with kleptomania experience displeasure at the thought of being caught in the act of the theft **(compare with Choice B)**. They do not want to get caught, as there is no associated thrill. The only pleasure felt is at the time of the actual stealing, and this is not related to the consequences of the act. This girl is unable to resist impulses to steal **(compare with Choice C)**. She may know on some level that stealing is morally wrong, as well as illegal, but she cannot override this insight because her impulse to steal is overwhelmingly strong. The quality of an increasing sense of tension immediately before the theft **(compare with Choice D)** is a necessary criterion for the diagnosis of kleptomania. Feeling pleasure at the time of committing the theft **(compare with Choice E)** is a necessary criterion for the diagnosis of kleptomania.
30
A 24-year-old, unemployed man is brought to the hospital by police after he was found sleeping on chairs at a departure gate in the airport. When awakened by the police and asked to leave the premises, the man insisted that he be allowed to stay in that specific location, since he was able to pick up on “special radiowave signals” emanating from that airline’s planes arriving between 1 PM and 3 PM. He spoke of being part of a secret mission with spies from an unidentified foreign country and stated that he was concerned that the police were working with his enemy. The police were able to calmly bring the man into the nearest psychiatric emergency department, where he was evaluated. He was taking no medications at the time, and a urine toxicology screen was negative. Which abnormality is most consistently seen in CT scans of the brains of persons who have the disorder suggested by the history and findings? (A) Atrophic changes in the cerebellar vermis (B) Bilateral hypodensities in the orbitofrontal region (C) Hydrocephalus (D) Increased ventricle-to-brain ratio (E) Significant sulcal widening
Respuesta: D The **correct answer is D**. Schizophrenia occurs in 1% of the population, equally affecting males and females. It is characterized by psychosis and disruption in one’s ability to function socially. Presenting complaints may include auditory hallucinations, strange belief systems, paranoia, lack of motivation, decrease in self-care, and peculiar mannerisms. In schizophrenia, neuropathologic volumetric analyses suggest a loss of brain weight, specifically of gray matter. CT studies may show a compensatory enlargement of the lateral and third ventricles, thereby increasing the ventricle-to-brain ratio. The temporal lobes appear to lose the most volume when compared with those of persons without schizophrenia. The frontal lobes may likewise have abnormalities; however, these are not related to the volume of the lobes but rather to the level of activity detected by functional MRIs. Atrophic changes in the cerebellar vermis **(Choice A)** are not typical for CT scans of patients with schizophrenia. The cerebellum is responsible for the regulation and control of muscular tone, the coordination of movement, and the control of posture and gait. Specifically, the vermis is the narrow middle zone between the two hemispheres of the cerebellum; it is unrelated to any dysfunction related to schizophrenia. Bilateral hypodensities in the orbitofrontal region **(Choice B)** are not typically seen in schizophrenic patients. Damage to the orbitofrontal region of the brain usually results in impaired social judgment. One criterion for the diagnosis of schizophrenia is social/occupational dysfunction. Often this is characterized by a disturbance in interpersonal relations. This is not thought to be due to an error in social judgment; likewise, it is not due to abnormalities in the orbitofrontal region of the brain. Hydrocephalus **(Choice C)** is a condition marked by an excessive accumulation of fluid, which leads to dilation of the ventricles and elevated intracranial pressure. There are several types of hydrocephalus, including but not limited to communicating, noncommunicating, normal pressure, and congenital. This condition is not related to schizophrenia. Significant sulcal widening **(Choice E)** is often seen as a result of atrophic changes consistent with aging and the dementias. Mild sulcal widening may be seen in scans of patients with schizophrenia, but significant changes are not characteristic.
31
A 59-year-old woman is brought to a psychiatrist because her family has been worried about her worsening behavior. The family reports prominent changes in personality and in the way in which she has recently been relating to others. They report that she has recently become sexually provocative in her attire and demeanor, she does not know how to hold socially appropriate conversations anymore, and she lacks the insight that she once had. She has a new grandchild and has recently become indifferent to him, and occasionally sits still, stares ahead, and seems mute. She is found to have mild memory and cognitive dysfunction, perseveration, and an inability to plan and organize. She has a positive glabellar sign. This patient most likely has dysfunction of which of the following regions of the brain? (A) Basal ganglia (B) Frontal lobe (C) Hypothalamus (D) Temporal lobe (E) Ventricular obstruction
Respuesta: B The **correct answer is B**. Frontal lobe dementia is characterized by damage to the frontal lobes and includes marked personality and behavioral changes as described in the question. The age of onset is most often between 50 and 60, and the condition is often progressive. Frontal lobe dementia is usually characterized by disproportionate impairment in tasks related to frontal lobe function, such as deficiency in abstract thinking, attentional shifting, or set formation. Disinhibition is also a key finding. CT or MRI reveals atrophy of the frontal lobe, especially early in the disease process. At present, the definitive diagnosis of any degenerative dementia is based on postmortem neuropathologic examination. Only one type of frontal lobe dementia, Pick disease, is associated with distinctive histopathologic abnormalities that allow for certain diagnosis. The patient’s glabellar sign is one of several signs elicited in a neurologic exam of a patient with frontal lobe dysfunction. The basal ganglia **(Choice A)** consist of deep subcortical nuclei responsible for movement disorders. The basal ganglia are composed of the caudate nucleus, putamen, globus pallidus, subthalamic nucleus, and substantia nigra. Certain movement disorders that result from basal ganglia dysfunction include Parkinson disease and hemiballism. The basal ganglia are not involved in this woman’s symptoms. The hypothalamus **(Choice C)** helps to maintain homeostasis through the secretion of hormones, central control of the autonomic nervous system, and the development of emotional and motivational states. The hypothalamus also interacts with limbic structures and the reticular formation for the maintenance of arousal. Temporal lobe dysfunction **(Choice D)** can occur in several ways; however, the resultant problems would be different. Temporal lobe seizures include simple partial seizures characterized by olfactory and gustatory hallucinations and complex partial seizures characterized by impairment of consciousness, repetitive psychomotor movements, and automatic behavior. Tumors in the temporal lobe may cause memory disturbances, superior quadrantanopsia, and, if the dominant temporal lobe is involved, aphasia. Disruption of the temporal lobe can involve limbic structures and tends to cause psychosis. Further, mania can be associated with temporal lobe lesions, especially on the right side. Ventricular obstruction **(Choice E)** results in hydrocephalus, which is an increase in the volume of cerebrospinal fluid within the skull. It can occur with or without an increase in pressure. Most commonly, there is an increase in pressure; this type of hydrocephalus can be divided into obstructive and communicating hydrocephalus. Many types of pathology can cause obstructive hydrocephalus, including brain tumors, inflammatory processes, and developmental abnormalities. Increased intracranial pressure may not be present in normal pressure hydrocephalus, which can cause a reversible dementia, but the characteristics of this dementia are not congruent with this woman’s symptoms.
32
A 34-year-old Caucasian woman presents to the emergency department with thoughts of suicide. She states that she was feeling “fine” until a week ago, when she began to feel very depressed. In addition, she states that she has not slept very much in the past few days and has been experiencing rapidly shifting extremes of mood, one day feeling “on top of the world” and the next feeling “tired and down in the dumps.” She claims that she has thoughts of jumping in front of a train to kill herself. When asked about illicit drug use, she becomes rather indignant, stating that she is being accused of being a drug addict. A urine drug screen is positive for cocaine metabolites. Which of the following is the most likely diagnosis? (A) Bipolar disorder, manic (B) Dysthymic disorder (C) Histrionic personality disorder (D) Major depressive disorder (E) Substance-induced mood disorder
Respuesta: E The **correct answer is E**. The key to this case is that the patient’s mood was “fine” until 1 week prior to presentation. Thereafter, her symptoms of mood lability and dysphoria with suicidal thoughts can be attributable to cocaine use and/or withdrawal. Once cocaine use stops, the mood symptoms generally improve within a few days, and suicidal ideation often resolves. In bipolar disorder, manic **(Choice A)**, elevated or irritable mood states generally last for days on end, even in so-called rapid cyclers. It would be difficult to make the diagnosis of bipolar disorder in this patient because of the positive urine drug screen and the fact that all of her symptoms can be ascribed to cocaine use. Dysthymic disorder **(Choice B)** is characterized by the presence of depressive symptoms that do not meet the full criteria for major depression, with a duration of at least 2 years. Rapidly shifting extremes of mood and acute suicidal impulses are not characteristic. Although individuals with histrionic personality disorder **(Choice C)** have rapidly shifting expressions of emotion and may have chronic dysphoria, the key feature of this disorder is the need and constant quest for attention from others. Major depressive disorder **(Choice D)** requires at least a 2-week history of a constellation of depressive symptoms. This diagnosis cannot be reliably made when symptoms can be otherwise explained, such as in this case of drug use.
33
A 33-year-old salesman returns from work following a meeting with his boss. The man demanded a raise because of his increased sales. He was told that there were other people who deserved it more. He comes back home and starts yelling at his son for watching TV after he had finished his homework. Which of the following most appropriately describes this man’s reaction? (A) Acting out (B) Displacement (C) Projection (D) Reaction formation (E) Sublimation
Respuesta: B The **correct answer is B**. In displacement, feelings that are otherwise unacceptable or dangerous are transferred or redirected onto a more acceptable (and usually less threatening) substitute object. In this way, their open expression is still allowed. Acting out **(Choice A)** refers to a defense mechanism by which the individual deals with unacceptable emotional conflict by actions rather than by reflection or expression of feelings. Projection **(Choice C)** is a defense mechanism that allows the individual to deal with emotionally conflicting issues by falsely attributing his or her own unacceptable thoughts, feelings, or impulses to another person. Reaction formation **(Choice D)** is a way individuals deal with emotional conflict by substituting diametrically opposed thoughts or feelings with their own unacceptable ones. Sublimation **(Choice E)** is a defense mechanism in which an individual deals with emotional conflict by channeling potentially maladaptive impulses or feelings into socially acceptable behaviors.
34
A 28-year-old Caucasian man is referred to a psychiatrist for management of anxiety. The patient reports that, in the past year, he has had “anxiety attacks,” during which he experiences an intense feeling of anxiety, chest pain, difficulty breathing, numbness and tingling in his hands and feet, and sweating. The attacks usually last for about 5 to 10 minutes and are unprovoked. The episodes used to occur only once or twice a month, but have been increasing in frequency to several times per day. He has been evaluated a number of times at an emergency department because he thought he was having a heart attack. Each emergency visit revealed no cardiac or other medical explanation for his symptoms. There is no family history of cardiac disease, and his physical examination is unremarkable. Which of the following is the most likely diagnosis? (A) Generalized anxiety disorder (B) Histrionic personality disorder (C) Hypochondriasis (D) Panic disorder (E) Somatization disorder
Respuesta: D The **correct answer is D**. Panic disorder is characterized by recurrent unprovoked panic attacks that have occurred for at least a month and cause significant distress in areas of daily functioning. Such patients often describe symptoms not dissimilar to a myocardial infarction, which obviously must be ruled out. The first-line treatment for panic disorder is a selective serotonin reuptake inhibitor. Benzodiazepines can be used to abort an attack. Alprazolam and clonazepam are benzodiazepines that are used for long-term management when SSRIs are ineffective or contraindicated, but carry with them the disadvantages of withdrawal and dependency, so long-term use is not recommended. Generalized anxiety disorder **(Choice A)** is characterized by excessive worry about several events or activities. Panic attacks are not the sole distressing symptom. An individual with histrionic personality disorder **(Choice B)** shows a pervasive pattern of excessive emotionality and attention seeking. He may exhibit an exaggerated expression of emotion; however, anxiety per se is not a defining feature. In hypochondriasis **(Choice C)**, there is the preoccupation with having a serious disease, but overt pathology is not evident. Although anxiety may be associated with this disorder, panic attacks typically are not. People with somatization disorder **(Choice E)** have various somatic complaints, which are chronic and do not occur in acute attacks.
35
A 23-year-old African American man with a diagnosis of schizophrenia, paranoid type, has been maintained on a low dose of haloperidol for the past 3 years. For the past month, his symptoms of paranoia and auditory hallucinations have been more prominent, and his psychiatrist decides to increase his dose of haloperidol. The patient’s mother calls the psychiatrist, concerned that he seems slower than usual and has a fine resting tremor of his upper extremities. Furthermore, he seems to be drooling and complains of being stiff. Which of the following is the most likely diagnosis? (A) Benign essential tremor (B) Neuroleptic-induced parkinsonism (C) Neuroleptic malignant syndrome (D) Parkinson disease (E) Tardive dyskinesia
Respuesta: B The **correct answer is B**. Neuroleptic-induced parkinsonism, which is clinically indistinguishable from Parkinson disease, occurs in the context of neuroleptic use. It is caused by decreased dopaminergic activity in the substantia nigra resulting from dopamine receptor antagonism. It will often appear in neuroleptic-naïve patients or in patients whose dose of neuroleptic has been increased, as in this case. In benign essential tremor **(Choice A)**, the only clinical feature is tremor. Other symptoms of parkinsonism, such as sialorrhea, stiffness, shuffling gait, and bradykinesia, are not features. Neuroleptic malignant syndrome (NMS) **(Choice C)** is characterized by rigidity, fever, altered mental status, and autonomic dysfunction. It is a rare complication of neuroleptic use. Other than rigidity, symptoms of parkinsonism are not present. Parkinson disease **(Choice D)** is idiopathic and often occurs late in life. It is clinically indistinguishable from drug-induced parkinsonism, but the fact that the presentation occurred in the context of an increased dose of haloperidol suggests drug-induced parkinsonism as the clear correct Choice. Tardive dyskinesia **(Choice E)** is a late complication of typical neuroleptic use. (Its association with the atypical neuroleptics has not been established.) It is a hyperkinetic movement disorder characterized by abnormal choreoathetoid movements often affecting the face and mouth. Parkinsonism, a hypokinetic movement disorder, is an entirely different entity.
36
A 48-year-old African American woman is brought to the emergency department by her mother because of "infestation". The patient states that she is infested with mites and points to areas on her arms. She reports that her house became infested with mites and other bugs about 3 months ago, and she has called an exterminator several times to have the house sprayed with a chemical. Further, she has been spraying herself with a pestcontrol agent daily because “the bugs are coming out of every part of my body, my eyes, my nose, and my mouth. I think my kids are getting infested, too!” She does not report auditory hallucinations. She denies any past psychiatric or medical history. Physical examination is remarkable for several scattered excoriated areas, but there is no evidence of infestation. Laboratory studies, including a urine drug screen, are negative. Which of the following is the most likely diagnosis? (A) Delirium tremens (B) Delusional disorder (C) Hypnagogic hallucination (D) Obsessive-compulsive disorder (E) Paranoid personality disorder (F) Schizophrenia
Respuesta: B The **correct answer is B**. This patient has delusional disorder of the somatic type. In this delusional disorder, there is the presence of a single delusion of having some general medical condition; other criteria for a primary psychotic disorder, such as schizophrenia, are not met. Hallucinations, if present, are not a primary feature and are congruent with the delusion (such as seeing the mites). This disorder tends to have an onset in middle age and responds poorly to treatment. Delirium tremens **(choice A)** is a severe withdrawal reaction that occurs in alcohol users following heavy ingestion and rapid reduction of alcohol consumption. Clinical features include delirium, sympathetic overactivity, electrolyte disturbances, and hallucinations that are commonly of the visual type. Delirium tremens can result in seizures, but improvement usually occurs in 36 hours. This patient has no history of alcohol use and displays a hallucination disorder that is over a long period of time (3 months). Hypnagogic hallucinations **(choice C)** describe hallucinations that occur when one is asleep. They are usually short, auditory hallucinations, such as a ringing bell, and are a feature of narcolepsy. In obsessive-compulsive disorder **(OCD; choice D)**, patients experience intrusive thoughts that are generally considered inappropriate and may feel compelled to engage in compulsions. The thoughts, although intrusive and inappropriate, are not delusional in nature, and hallucinations are clearly not a feature. Individuals with paranoid personality disorder **(choice E)** have pervasive mistrust toward others; however, somatic delusions and hallucinations are not present. Schizophrenia **(choice F)** is incorrect. It would be highly unlikely for its onset to be in the fifth decade, whereas this would not be unusual for delusional disorder. In addition, to make the diagnosis of schizophrenia, symptoms need to present for at least 6 months.
37
A 33-year-old man being treated for bipolar disorder develops gradual slowing of cognition, with concomitant development of a gravelly voice, constipation, dry skin, and hair loss. Examination is normal, except for a lag in the relaxation phase of the ankle jerk reflex. His only medication is lithium, and a lithium level is within normal limits. Which of the following tests should most likely be ordered next? (A) BUN and creatinine (B) Liver function tests (C) Serum sodium (D) Thyroid function tests (E) Urine electrolytes
Respuesta: D The **correct answer is D**. This patient has several classic symptoms of hypothyroidism. Lithium inhibits secretion by the thyroid gland and may result in hypothyroidism and a nontoxic goiter. Decreased secretion of thyroid hormone should be reflected by low levels of thyroid hormone, and by an increased serum TSH. This can be treated with thyroid replacement and does not necessarily mean that lithium should be discontinued. BUN and creatinine **(choice A)** may be elevated secondary to any one of a number of renal complications from lithium treatment, including diabetes insipidus (see explanation for choice C below), interstitial fibrosis, or focal renal necrosis. With the exception of slowed cognition, however, the patient’s symptoms are not suggestive of uremia. Liver function tests **(choice B)** are not frequently disrupted by lithium therapy and, even if elevated, would not produce the clinical picture described in this case. Serum sodium **(choice C)** may be elevated in patients taking lithium who develop diabetes insipidus (DI). Lithium causes a nephrogenic form of DI in which the renal tubules do not respond to antidiuretic hormone, with resultant loss of free water. Hypernatremia may cause mental status changes but none of the other symptoms. Urine electrolytes **(choice E)** would also be helpful in making a diagnosis of nephrogenic DI. In DI, the urine electrolytes should be diluted by the excess free water in the urine. Symptoms of DI will usually remit with discontinuation of lithium or sometimes simply with decreased dosage.
38
A 30-year-old woman comes to her physician with multiple complaints. She has had insomnia, loss of appetite, and anxiety for 6 months. She also reports intestinal symptoms, such as epigastric pain and bloating. She has thought of quitting her work as a secretary because her husband wants her to stay at home to take better care of their 3-year-old boy. Physical examination reveals multiple bruises on her trunk. On further questioning, the patient admits that she has been physically abused by her husband. The violence has been increasing in frequency and severity over the past few months. Which of the following is the most appropriate next step in short-term management? (A) Express doubts about the patient’s account but document lesions carefully (B) Express indignation against the perpetrator (C) Instruct the patient on how and when to leave her husband (D) Provide emotional support and ensure the patient’s safety (E) Refer both the victim and her husband for marriage counseling (F) Try to persuade the patient’s husband to quit abusing his wife
Respuesta: D The **correct answer is D**. The prevalence of domestic violence (partner or spousal abuse) is high. Approximately 12 to 28% of women report having suffered physical abuse by their husbands or partners, according to recent statistics. Although domestic violence is a definite medical diagnosis (code 995.81 of the International Classification of Disease, 9th ed.), it is often unrecognized by primary care physicians. Most victims of domestic violence come to clinical attention because of seemingly unrelated problems, including anxiety, depression, and various physical complaints. Identification of cases of domestic violence relies on two general strategies: universal screening (if generic questions on domestic violence are asked in the context of history taking) and case finding (when specific evidence of physical abuse is observed during physical examination). Once a victim of domestic violence has been identified, such as in this example, the next steps include providing emotional support and information about social resources, ensuring safety, documenting injuries carefully, and scheduling follow-up appointments. There are a number of mistakes that should be avoided: Expressing doubts about the patient’s account **(choice A)** will most likely cause the victim to cease seeking help. Furthermore, studies have shown that the great majority of victims’ accounts regarding domestic violence are accurate. Expressing indignation against the perpetrator **(choice B)** may result in losing rapport with the victim. Usually, a victim of spousal abuse has a conflicting relationship with the perpetrator of violence. She both loves and fears him, and the physician’s outrage may offend her. Instructing the patient on how and when to leave her husband **(choice C)** is a mistake. Victims are at greatest risk of suffering injury and death precisely when they try to leave a relationship with the abuser. Referring both the patient and her husband for marriage counseling **(choice E)** is another common mistake. Referral to counseling implies breaching of confidentiality and may expose the victim to increased abuse. Trying to persuade the patient’s husband to stop abusing her **(choice F)** should never be attempted, even if the victim expresses consent. This action entails breaching of confidentiality and usually results in retaliatory abuse by the perpetrator.
39
A 45-year-old, obese woman presents to a psychiatrist with depression. She describes two previous similar episodes that occurred in the past 2 years that were milder and resolved with the support of her family. Her symptoms are much worse now, and she is prepared to do anything to help herself. On the basis of her information, the psychiatrist decides to start her on phenelzine for treatment of atypical depression. Which of the following features was likely present in the patient’s history to lead to the diagnosis of major depressive disorder with atypical features? (A) Decreased appetite (B ) Depression worse in the morning and better in the evening (C) Leaden paralysis (D) Negativism (E) Psychomotor agitation
Respuesta: C The **correct answer is C**. In major depressive disorder with atypical features, mood reactivity is a response to potential or actual positive events, with at least two significant symptoms present: significant weight gain or increase in appetite (cravings for sweets at times), hypersomnia, heavy feeling in arms and legs (“leaden paralysis”), and increased sensitivity to interpersonal rejection resulting in social dysfunctioning. Decreased appetite **(choice A)** and weight loss are more common, but not specific, features of several psychiatric disorders and medical conditions. They are not features of atypical depressive disorder. Worsening of depression in the morning with some improvement of mood in the evening **(choice B)** is typically seen in major depressive disorder with melancholic features and is defined as diurnal variation of mood. Negativism **(choice D)** is manifested by maintenance of rigid posture or resistance to commands and instructions. It can be seen as a part of normal childhood development and in a variety of psychiatric disorders. Psychomotor agitation **(choice E)** is a symptom that can be seen in several acute psychiatric disorders, including affective disorders, psychotic disorders, substance-induced disorders, and cognitive disorders like delirium and dementia.
40
A 63-year-old woman with a 9-year history of Parkinson disease has developed worsening problems with visual hallucinations. In particular, she complains of seeing cats crawling along the floors in her house. She is taking L-dopa/carbidopa (Sinemet) and has had significant improvement in her rigidity. Which of the following drugs would be most appropriate for her psychosis? (A) Chlorpromazine (B) Clomipramine (C) Clozapine (D) Haloperidol (E) Pergolide
Respuesta: C The **correct answer is C**. All the new atypical antipsychotics have reduced extrapyramidal side effects, and clozapine has the least effect on the basal ganglia. It works predominantly as an antagonist to D1, D3, and D4 dopamine receptors. Of all the antipsychotics, it has the lowest activity against D2 receptors, which is the best measure of a drug’s potential to cause extrapyramidal side effects. Clozapine appears more effective in the mesolimbic dopamine pathways, which are disrupted in psychotic states and less effective in the nigrostriatal systems crucial to fluid movement. The main limitation to using clozapine is the risk of agranulocytosis (1 to 2%), which requires weekly leukocyte counts for the first 6 months of treatment, then every 2 weeks thereafter. Chlorpromazine **(choice A)** is an example of a typical antipsychotic. It is considered a low-potency antipsychotic and has less of a tendency to cause extrapyramidal side effects than do the high- potency antipsychotics. It is still more likely than any of the atypical antipsychotics to worsen this patient’s parkinsonism and should be avoided here. Clomipramine **(choice B)** is a tricyclic antidepressant. In addition to its inhibition of norepinephrine reuptake, it is a strong serotonin reuptake inhibitor, making it a good choice for treatment in obsessive-compulsive disorder (OCD) when SSRIs are poorly tolerated. It has no role in the treatment of psychosis. Haloperidol **(choice D)** is one of the high-potency anti-psychotics, and it may produce prominent extrapyramidal side effects. It would very likely improve this patient’s psychosis but would also worsen her parkinsonian symptoms. In general, haloperidol is never used in patients with Parkinson disease. Pergolide **(choice E)** is a dopamine agonist and may be used as adjuvant therapy in Parkinson disease to lessen the daily Sinemet requirement. It would tend to produce the same CNS side effects as Sinemet, including dyskinesias and visual hallucinations.
41
A 29-year-old Caucasian man is brought into the emergency department by ambulance. He was found unresponsive in an alleyway by a passerby. On arrival to the emergency department, the patient is responsive to deep pain only, respirations are slow and shallow, and the pupils are pinpoint. There are track marks noted on the upper and lower extremities. Which of the following is the most appropriate next step in management? (A) Chlordiazepoxide (B) Flumazenil (C) Methadone (D) N-Acetylcysteine (E) Naloxone (F) Naltrexone
Respuesta: E The **correct answer is E**. Overdose of opiates, such as heroin, causes CNS and respiratory depression and miosis. Naloxone is a short-acting opioid antagonist that reverses the signs and symptoms of opiate overdose. Because it has a short half-life, it may need to be administered repeatedly. Chlordiazepoxide **(choice A)** is a long-acting benzodiazepine that is used in treatment of alcohol withdrawal. It would likely exacerbate this patient’s condition. Flumazenil **(choice B)** is a benzodiazepine antagonist that can reverse respiratory and CNS depression from severe benzodiazepine overdose. Methadone **(choice C)** is a synthetic compound with a similar action to that of morphine and heroin, and it is almost equal to them in addiction liability. It is used as a method of long-term opiate withdrawal by monitored administration and gradual tapering. It, however, has no effect in the immediate management of opiate overdose. N-Acetylcysteine **(NAC; choice D)** protects against liver damage in early acetaminophen poisoning by production of cysteine, which acts as a glutathione precursor. NAC also acts by supplying additional thiol groups that bind directly with the reactive metabolites of acetaminophen. It has no place in the therapy of opiate overdose. Naltrexone **(choice F)** is also an opioid receptor antagonist; however, it is the incorrect choice because its onset of action is too slow to be useful in acute overdose. It has indication in the treatment of both alcohol and opiate dependence to assist with craving.
42
A 22-year-old woman with a history of depression is brought to the emergency department by her friend, who found her groggy and minimally responsive at home. On examination, she is mildly obtunded and complains of blurry vision. She has enlarged, minimally reactive pupils, but no focal deficits. A cardiac monitor demonstrates frequent premature ventricular contractions and tachycardia to 120 beats per minute. A Foley catheter is inserted, with passage of 800 mL of urine. An overdose is presumed. Which of the following drugs did the patient most likely take? (A) Bupropion (B) Fluoxetine (C) Nortriptyline (D) Phenelzine (E) Sertraline
Respuesta: C The **correct answer is C**. Nortriptyline is a tricyclic antidepressant. Like most tricyclics, it has prominent anticholinergic properties, which would explain the patient’s lethargy, blurry vision with mydriasis, urinary retention, and tachycardia. The tricyclics may be lethal in overdose because of their tendency to cause arrhythmias; therefore, the patient should continue to be observed on a cardiac monitor. Bupropion **(choice A)** is a second-generation antidepressant without anticholinergic side effects. It can cause seizures in up to 0.5% of patients when taken at high doses. It is therefore contraindicated in patients with epilepsy. Fluoxetine **(choice B)** and sertraline **(choice E)** are selective serotonin reuptake inhibitors (SSRIs). This class of antidepressants is considered relatively safe in overdose. It also has only minimal anticholinergic effects. One side effect that limits compliance with this class of drugs is sexual dysfunction. Phenelzine **(choice D)** is a monoamine oxidase inhibitor. This class of drug may induce a life-threatening hypertensive crisis when patients eat foods containing tyramine. They may also precipitate a serotonin syndrome when taken in excess or combined with an SSRI.
43
A 19-year-old man presents to the emergency department after a car accident in which he was driving drunk. He was wearing a seat belt and sustained only minor cuts and bruises. When questioned about his history of drinking alcohol, he states that he has driven while intoxicated before and that his drinking has caused him to miss work on a few occasions. He was arrested twice for driving under the influence (DUI). He says that his drinking caused his parents to force him out of the house last month. When questioned further, he states that in the past it took him a few shots of vodka to get drunk, but now it takes up to a pint of liquor. Which of the following factors in the patient’s history suggests a diagnosis of substance dependence rather than substance abuse? (A) Continued substance use despite persistent familial problems (B) Failure to fulfill work obligations because of substance use (C) Increased amount of alcohol necessary to achieve intoxication (D) Recurrent alcohol-related legal problems, like a DUI arrest (E) Recurrent substance use while driving
Respuesta: C The **correct answer is C**. The fact that it takes more alcohol to achieve intoxication is indicative of tolerance, a diagnostic criterion that makes substance dependence more likely than substance abuse. Any history of withdrawal from a substance also more strongly suggests substance dependence. **Choices A-D** are all DSM-IV criteria for substance abuse, provided they occur within a 12-month period and there is no indication of tolerance or withdrawal.
44
A 31-year-old woman has had panic attacks for almost 15 years. She recalls that they first occurred when she was sitting in large crowds at her high school’s sporting arenas. She remembers feeling trapped on the football bleachers and feeling that her world was closing in on her. Her heart began racing, and she became sweaty and had to lay down to avoid fainting. She thereafter stopped attending large events; however, the panic attacks resumed when she went to college and had to wait in long cafeteria lines. After college, she became very nervous traveling on both the subways and public buses to get to work. She had several panic attacks and opted to quit her job and find another one within walking distance from her home. Shortly thereafter, she began having panic attacks entering her new work office building. These were unprecedented, uncontrollable, and very frightening to her. She left this job and for the past 3 years has not left her home. She allows her family to visit her, bring her food, and do errands for her, but she refuses to leave her house. Which of the following behavioral therapies has been shown to most effectively treat this condition? (A) Biofeedback (B) Flooding (C) Gradual exposure (D) Learning new habits (E) Relaxation therapy
Respuesta: C The **correct answer is C**. This patient has panic disorder with agoraphobia, which is characterized by anxiety about being in places or situations from which escape may be difficult or in which help may not be available in the event of having an unexpected or situationally predisposed panic attack. Agoraphobia itself is a fear of open spaces. Gradual exposure is often used for agoraphobia, specific phobia, and obsessive-compulsive disorder. It involves exposure to situations, specifically open spaces for patients with agoraphobia, organized along a hierarchy of gradually more fear- inducing situations. It is usually performed in small steps at a time, with the experiencing of anticipatory anxiety in smaller, more tolerable exposures. The patient is monitored and in the presence of the therapist throughout the gradual exposure. Biofeedback **(choice A)** involves the recording and display of small changes in the feedback parameter. The display may be visual, such as a bar of lights, or auditory. Patients are instructed to change the levels of the parameter, using the feedback from the display as a guide. The treatment was based on the discovery that laboratory animals could control their autonomic function by using biofeedback under stringent conditions that appeared to eliminate possible mediation by skeletal muscle activity. On the basis of these findings, biofeedback has been used with certain patients, such as those with urinary incontinence, who can use biofeedback to regain control over their pelvic musculature. It can be used by itself or in conjunction with relaxation and breathing retraining. Flooding **(choice B)** involves exposure to a maximally fear-inducing situation, which can be real or contrived. It may seem traumatizing, but usually flooding therapy sessions are long and consist of continually decreasing levels of anxiety. Learning new habits **(choice D)** is useful in many preventive health behaviors that ought to become regular occurrences. This is more practical in behaviors such as using seat belts, brushing teeth, and taking prescribed medications. To build a new habit, one can identify regularly occurring behaviors and use the stimuli from these behaviors to prompt the new habit. This is not a way to help with agoraphobic patients, however. Relaxation therapy **(choice E)** is used either as a component of treatment programs (i.e., biofeedback or systematic desensitization) or as its own treatment. It is characterized by immobility of the body, control over the focus of attention, low muscle tone, and cultivation of a specific, contemplative frame of mind. If done properly, muscle tension, respiratory rate, heart rate, and blood pressure decrease. Indications are for any conditions thought to be related to adrenergic stress responses. Patients with agoraphobia may find some benefit in relaxation therapy, but alone it will not treat the fear of open spaces.
45
A 20-year-old woman is brought to the emergency department by paramedics. She is very anxious and appears frightened; her speech is slurred and dysarthric. She is extremely labile emotionally, and she has been thrashing about and scraping herself inadvertently, requiring restraint. Her blood pressure is 180/95 mm Hg, and her pulse is 120/min. On examination, she has horizontal nystagmus in both eyes. Which of the following is the most likely diagnosis? (A) Alcohol withdrawal (B) Lithium intoxication (C) Neuroleptic malignant syndrome (D) Paranoid schizophrenia (E) Phencyclidine intoxication
Respuesta: E The **correct answer is E**. The patient’s symptoms of hypertension, tachycardia, nystagmus, dysarthria, impulsiveness, and psychomotor agitation are all indicative of phencyclidine intoxication. Alcohol withdrawal symptoms **(choice A)** do not include nystagmus. Instead, tremor, seizures, or hallucinations are common. Lithium intoxication **(choice B)** is associated with symptoms of coarse tremor, impaired consciousness, and, most importantly, myoclonus, seizures and coma. Nystagmus is not associated with lithium toxicity. Neuroleptic malignant syndrome **(choice C)** is a life-threatening complication of antipsychotic treatment. The symptoms include muscular rigidity and dystonia, akinesia, mutism, obtundation, and agitation. The autonomic symptoms include high fever, sweating, and increased blood pressure. This patient does not have fever, nor is there any indication of antipsychotic use. Paranoid schizophrenia **(choice D)** is a chronic condition characterized by frequent auditory hallucinations of more than 6 months’ duration. The acute nature of this patient’s symptoms and her physical findings make this an unlikely diagnosis.
46
An 83-year-old woman is brought from her nursing home residence to the local psychiatric emergency evaluation center. She was diagnosed with Alzheimer type dementia 4 years ago. She is described as usually being calm and pleasantly confused (disoriented to place, date, and time). She had been stable, but for the past 3 nights she woke up in the middle of the night, shouting out incoherent statements and apparently hallucinating. In the daytime of the past 3 days, she has been eating less and occasionally napping (which is unusual for her), with episodes of agitation during which she has tried to hit other nursing home residents. She has also had a low-grade fever. In the psychiatric emergency evaluation center, she appears excited and is asking for her husband, who has been deceased for 12 years. Which of the following is the most appropriate initial step in management? (A) Inquire about her medications, including any recent changes (B) Obtain vital signs and an ECG (C) Protect the patient from unintentional harm (D) Request a medicine consult (E) Administer intramuscular haloperidol
Respuesta: C The **correct answer is C**. This patient with dementia probably also has delirium. Dementia, delirium, amnestic disorder, and other cognitive disorders are usually grouped together under the category of organic mental disorders. The term “organic” implies brain dysfunction. The patient’s change in mental status from being calm and pleasant to being disruptive and agitated is indicative of a likely delirium. Also, a low-grade fever in an elderly person should not be taken lightly. It is often more difficult for the elderly to mount a substantial fever, so any temperature above normal is considered abnormal and may be serious. By definition, delirium is an acute reversible mental disorder characterized by confusion and some impairment of consciousness. It is usually associated with emotional lability, hallucinations (more commonly visual than auditory), and often violent behavior. The most important first step in treating this patient is protecting her from unintentional harm. It would be quite unfortunate for this patient to hurt herself in the context of an acute delirium. Calming her with words, holding her hands, removing potentially dangerous objects, and possibly putting her in a restrictive posey vest, if necessary, should be the first steps taken to ensure this patient’s safety. Inquiring about this woman’s medications, including any recent changes **(choice A)**, is very important and should be among the initial steps in history gathering after the patient is restricted from hurting herself and is medically and psychiatrically evaluated. Often the patient herself will be unable to report this information, and a treating staff member from the nursing home should be contacted. Obtaining vital signs and an ECG **(choice B)** is important, but should be done after safety measures are taken. Any changes in vital signs or in an ECG from the patient’s baseline should be carefully attended to and treated as necessary. It may be necessary to request a medicine consult **(choice D)**, but this can be determined only after the patient’s condition is understood and evaluated better. Safeguarding the patient, evaluating her condition, and determining the problems at hand should be performed prior to any thought of asking for a medicine consult. Administering intramuscular haloperidol **(choice E)** is too aggressive to be taken as a first step. It is better to avoid medications in this patient to get a clearer picture of what her presentation is really like and also to avoid further sedation. Moreover, the elderly tend to be very sensitive to medications. If haloperidol were required, it would be judicious to use the smallest dose possible.
47
A 26-year-old man claims that he was smoking crack cocaine at a party and became intensely paranoid. He thought that his mother was trying to kill him and that his girlfriend was plotting to have him imprisoned for treason against the U.S. In addition, he experienced the sensation of bugs crawling under his skin. On examination, the patient appears somewhat anxious, but is able to make himself well understood. He currently denies auditory or visual hallucinations, and the paranoia, albeit present, is milder and less distressing. Physical examination is unremarkable, and laboratory studies are significant only for a urine drug screen positive for cocaine metabolites. Overactivity at which of the following receptors best explains this patient’s presenting symptoms? (A) Acetylcholine (B) Dopamine (C) GABA (D) Glutamate (E) Glycine
Respuesta: D The **correct answer is B**. Cocaine blocks dopamine reuptake at the dopamine transporter, increasing dopamine concentration at the D1 and D2 receptors, which appear to mediate many of the symptoms of psychosis. The dopamine system is also implicated in schizophrenia, which is characterized by psychotic symptoms, such as hallucinations and delusions. Medications that block acetylcholine **(choice A)** are used in psychiatry for neuroleptic-induced movement disorders. Furthermore, acetylcholinesterase inhibitors, such as donepezil, are used in Alzheimer type dementia. Benzodiazepines and barbiturates are sedative-hypnotics that act at GABA **(choice C)** receptors. These drugs are not associated with psychosis. If the patient admitted to phencyclidine (PCP) use, and PCP were detected on the urine drug screen, glutamate might be an appropriate answer. PCP inhibits the NMDA receptor, which uses glutamate **(choice D)** as a neurotransmitter. Given the information, cocaine is the most likely drug involved; therefore, choice B is correct. Glycine receptors **(choice E)** are largely located in the spinal cord. Strychnine, a glycine antagonist, causes convulsions.
48
A 77-year-old woman is admitted to the hospital late in the afternoon for the treatment of urosepsis. The on-call resident is paged in the middle of the night to evaluate the patient because of a mental status change. The patient is heard yelling that she is in prison and states that she is covered with insects. Her primary nurse says that the patient has been alternating between periods of overt agitation and lethargy over the past 12 hours since she has been on the ward. Which of the following is the most likely diagnosis? (A) Amnestic disorder not otherwise specified (B) Delirium (C) Major depressive disorder, with psychotic features (D) Parkinson dementia (E) Senile dementia, Alzheimer type
Respuesta: B The **correct answer is B**. The patient has a mental disturbance characterized by alterations in cognition, attention, alertness, and perception. Delirium is, by definition, caused by a separate physiologic condition such as illness or substance intoxication/withdrawal. Essential features of the diagnosis of delirium include the development of symptoms over a short period of time and a fluctuating clinical presentation. Amnestic disorders **(choice A)** are conditions in which there is gross impairment in memory, but level of alertness and other cognitive problems are not evident. The most striking feature of this patient’s presentation includes disturbances in cognition and perception. The patient in question evidences no depressive complaints and the abrupt production of symptoms and their waxing and waning course are not consistent with a depression. Therefore, **choice C** is incorrect. The patient does not have a known history of Parkinson disease and there is no report of the neurologic symptoms of Parkinson disease in this patient. A diagnosis of Parkinson dementia is not tenable in this patient. Therefore, **choice D** is incorrect. The diagnosis of senile dementia, Alzheimer type **(choice E)** requires an extensive history of difficulties in memory, language, motor behaviors, recognition of people or objects, and disturbances in executive functioning. Alzheimer dementia cannot be diagnosed in the context of a delirium.
49
A 29-year-old Caucasian man visits an emergency department complaining that “black women are putting spells on me and are trying to rape me.” He reports that for the past 6 years he has had held the firm belief that women he encounters are hypnotizing him and then raping him. The patient also states that he can hear voices of women in his head talking “dirty” to him. His mental status examination is remarkable for the delusions and hallucinations noted above. His thought process is goal-oriented, and his affect is constricted. Physical examination and laboratory studies, including a drug screen, are unremarkable. Which of the following is the most likely diagnosis? (A) Bipolar disorder (B) Brief psychotic disorder (C) Major depressive disorder with psychotic features (D) Posttraumatic stress disorder (E) Schizophrenia
Respuesta: E The **correct answer is E**. The presence of delusions of persecution and auditory hallucinations are the hallmarks of paranoid type schizophrenia. The disorder requires symptoms that meet criteria for schizophrenia be present for greater than 6 months (psychotic symptoms themselves need be present for only 2 weeks, incidentally); therefore, brief psychotic disorder **(choice B)**, in which psychotic symptoms resolve between 1 day and 1 month, is incorrect. Although individuals with bipolar disorder **(choice A)** may have delusions, they are usually but not always of a grandiose quality. Furthermore, an individual with acute mania would have a thought process marked by loosening of associations or flight of ideas. In addition, the affect of a bipolar patient would likely be rather animated, not constricted, as in this case. Major depressive disorder with psychotic features **(choice C)** is incorrect because the psychotic symptoms associated with this disorder tend not to be bizarre, as they are in this individual. It is also a time-limited disorder and would not endure for an entire 6 years without some degree of remission. If this had been the correct answer, there should have been other symptoms of depression. Although rape is a recognized stressor that may predate posttraumatic stress disorder **(PTSD; choice D)**, the preoccupation in this case is clearly delusional since the patient reports that women are putting spells on him. In addition, auditory hallucinations are not a feature of PTSD. Again, one would look for other classic symptoms of PTSD, such as hypervigilance, increased startle reflex, and avoidance behavior, to make the diagnosis.
50
A forensic psychiatrist is evaluating a patient before a criminal trial. The suspect is a 17-year-old girl who is accused of fatally stabbing her stepfather while he was asleep. There is substantial forensic evidence against the suspect, including a murder weapon with the suspect’s fingerprints. The suspect states that she does not remember stabbing her stepfather. However, she feels no sense of loss that her stepfather is dead because of a long history of being sexually abused by him. During the assessment, the suspect’s facial expression and voice rapidly change. She says that she is now the “adopted older brother” of the suspect and that he stabbed the suspect’s stepfather without her knowledge. For the remainder of the interview, the psychiatrist is unable to speak to the suspect directly. Which of the following is the most likely diagnosis? (A) Depersonalization disorder (B) Dissociative fugue (C) Dissociative identity disorder (D) Major depressive disorder, with psychotic features (E) Schizophrenia, paranoid type
Respuesta: The **correct answer is C**. Patients with dissociative identity disorder manifest multiple distinct personalities that assume control of the patient’s awareness of the environment and behavior. Common comorbidities for patients with dissociative identity disorder include borderline personality disorder, posttraumatic stress disorder, and physical or sexual abuse. Depersonalization disorder **(choice A)** is defined by the perception that one is detached from his/her mental processes or body. It is not characterized by the assumption of multiple personalities. Dissociative fugue **(choice B)** is a rare dissociative disorder involving abrupt travel and an amnesia for events that may include the circumstances preceding arrival at one’s destination and one’s identity. The etiology of dissociative fugue is unknown, though a possible correlation between dissociative fugue and recent life stressors may exist. The condition is usually self-limited. This patient does not report symptoms consistent with a major depressive disorder **(choice D)**, such as depressed mood or neurovegetative signs of depression. The patient in question has periods of dissociation involving the assumption of another personality. This is not to be confused with a chronic psychotic state such as found in schizophrenia, chronic paranoid type **(choice E)**. Patients with schizophrenia typically evidence a disorganization of thought process and reality testing mechanisms that are inadequate. Patients with dissociative identity disorder have a fragmentation of actual conscious functions with intact reality testing.